vi the chicago school andschool and … · 381 s e c t i o n vi the chicagothe chicago school...

56
381 S E C T I O N VI THE CHICAGO THE CHICAGO SCHOOL AND SCHOOL AND CULTURAL/ CULTURAL/ SUBCULTURAL SUBCULTURAL THEORIES OF THEORIES OF CRIME CRIME T he introduction to this section will examine the origin and evolution of the Chicago/Ecological School theory, otherwise known as the ecological perspective or the theory of social disorganization. We will also discuss modern research on this theory, which assumes that the environment people live in determines their behav- ior. Finally, the assumptions and dynamics of cultural/subcultural theory in society will be discussed, with an emphasis on differences in certain models emphasizing inner-city subcultures and other modern examples (e.g., street gangs). We will finish this section introduction by reviewing the policy implications that have been suggested by this per- spective of crime.

Upload: lamnhu

Post on 12-Apr-2018

217 views

Category:

Documents


2 download

TRANSCRIPT

381

S E C T I O N

VITHE CHICAGOTHE CHICAGOSCHOOL ANDSCHOOL ANDCULTURAL/CULTURAL/SUBCULTURALSUBCULTURALTHEORIES OFTHEORIES OFCRIMECRIME

The introduction to this section will examine the origin and evolution of theChicago/Ecological School theory, otherwise known as the ecological perspectiveor the theory of social disorganization. We will also discuss modern research on

this theory, which assumes that the environment people live in determines their behav-ior. Finally, the assumptions and dynamics of cultural/subcultural theory in society willbe discussed, with an emphasis on differences in certain models emphasizing inner-citysubcultures and other modern examples (e.g., street gangs). We will finish this sectionintroduction by reviewing the policy implications that have been suggested by this per-spective of crime.

yy IntroductionThis section examines the Chicago School of criminology, which is otherwise known asthe ecological perspective or theory of social disorganization, for reasons that will becomevery clear. The Chicago School evolved there because the city at that time (late 19th andearly 20th centuries) desperately needed answers for its exponentially growing problemof delinquency and crime. Thus, this became a primary focus in the city of Chicago,where total chaos prevailed at the time.

A significant portion of the Chicago perspective involved the transmission of culturalvalues to other peers, and even across generations, as the older youths relayed their anti-social values and techniques to the younger children. Thus, the cultural/subcultural per-spective is also a key area of this theoretical model. This cultural aspect of the Chicagomodel is also examined in this section, as well as other subculture frameworks of offend-ing behaviors.

yy The School of Ecology andthe Chicago School of Criminology

Despite its name specifying one city, the Chicago School of criminology represents oneof the most valid and generalizable theories we will discuss in this book, in the sense thatmany of its propositions can be readily applied to the growth and evolution of virtuallyall cities around the world. The Chicago School, which is often referred to as theEcological School or the theory of social disorganization, also represents one of the ear-liest examples of balancing theorizing with scientific analysis and at the same time guid-ing important programs and policy implementations that still thrive today. Perhaps mostimportant, the Chicago School of criminology was the epitome of using theoretical devel-opment and scientific testing to help improve conditions in society when it was mostneeded, which can be appreciated only by understanding the degree of chaos and crimethat existed in Chicago in the late 1800s and early 1900s.

Cultural Context: Chicago in the 1800s and Early 1900s

Experts have determined that 19th-century Chicago was the fastest-growing city inU.S. history.1 Census data show the population went from about 5,000 in the early 1800sto more than 2 million by 1900; put another way, the population more than doubledevery decade during the 19th century.2 This massive rate of growth—much faster thanthat seen in other large U.S. cities such as Boston, Baltimore, New York, Philadelphia, andSan Francisco—was due to Chicago’s central geographic position. It was in many waysland-locked because although it sits on Lake Michigan, there was no water route to thecity from the Atlantic Ocean until the Erie Canal opened in 1825, which provided accessto the Great Lakes region for shipping and migration of people. Three years later came

382 CRIMINOLOGICAL THEORY: A TEXT/READER

1For an excellent discussion of the early history of Chicago, see Thomas J. Bernard, The Cycle of Juvenile Justice (NewYork: Oxford University Press, 1992).2See discussion in George B. Vold, Thomas J. Bernard, and Jeffrey B. Snipes, Theoretical Criminology, 5th ed. (NewYork: Oxford University Press, 2002), 117–22.

Section VI � The Chicago School and Cultural/Subcultural Theories of Crime 383

the first U.S. passenger train, the Baltimore & Ohio railroad, with a route from a mid-Atlantic city to central areas. These two transportation advancements created a continu-ous stream migration to the Chicago area, increased again when the transcontinentalrailroad was completed in 1869, linking both coasts with the U.S. Midwest.3

It is important to keep in mind that in the early to mid-1800s, many large U.S. citieshad virtually no formal social agencies to handle problems of urbanization: No socialworkers, building inspectors, garbage collectors, or even police officers. Once police agen-cies were introduced, their duties often included finding lost children and collecting thegarbage, primarily because there weren’t other agencies to perform these tasks. Therefore,communities were largely responsible for solving their own problems, including crime anddelinquency. By the late 1800s, however, Chicago was largely made up of citizens who didnot speak a common language and did not share each other’s cultural values. This phe-nomenon is consistent with Census Bureau data from that era, which shows that 70% ofChicago residents were foreign born and another 20% were first-generation Americans.Thus, it was almost impossible for these citizens to organize themselves to solve commu-nity problems because in most cases, they could not even understand each other.

This resulted in the type of chaos and normlessness that Durkheim predicted wouldoccur when urbanization and industrialization occurred too rapidly; in fact, Chicago rep-resented the archetypal example of a society in an anomic state, with almost a completebreakdown in control. One of the most notable manifestations of this breakdown insocial control was that children were running wild on the streets in gangs, with adultsmaking little attempt to intervene. So delinquency was soaring, and it appeared that thegangs controlled the streets as much as any other group.

The leaders and people of Chicago needed theoretical guidance to develop solutions totheir problems, particularly regarding the high rates of delinquency. This was a key factor inwhy the Department of Sociology at the University of Chicago became so important anddominant in the early 1900s. Essentially, modern sociology developed in Chicago becausethe city needed it the most to solve its social problems. Thus, Chicago became a type of lab-oratory for the sociological researchers, and they developed a number of theoretical mod-els of crime and other social ills that are still shown to be empirically valid today.

Ecological Principles in City Growth and Concentric Circles

In the 1920s and 1930s, several new perspectives of human behavior and city growthwere offered by sociologists at the University of Chicago. The first relevant model wasproposed by Robert E. Park, who claimed that much of human behavior, especially theway cities grow, follow the basic principles of ecology that had been documented andapplied to wildlife for many years at that point.4 Ecology is essentially the study of thedynamics and processes through which plants and animals interact with the environ-ment. In an application of Darwinian theory, Park proposed that the growth of citiesfollows a natural pattern and evolution.

3These dates were taken from The World Almanac, 2000, Millennium Collector’s Edition, (Mahwah, NJ: PrimediaReference, 2000).

4Robert E. Park, “Human Ecology,” American Journal of Sociology 42 (1936); Robert E. Park, Human Communities(Glencoe, IL: The Free Press, 1952).

Specifically, Park claimed that cities represent a type of complex organism with asense of unity composed of the interrelations among the citizens and groups in the city.Park applied the ecological principle of symbiosis to explain the dependency of variouscitizens and units on each other: Everyone is better off working together as a whole.Furthermore, Park claimed that all cities would contain identifiable clusters, which hecalled natural areas, where the cluster had taken on a life or organic unity by itself. Toclarify, many cities have neighborhoods that are made up of primarily one ethnic groupor are distinguished by certain features. For example, New York City’s Hell’s Kitchen,Times Square, and Harlem represent areas of one city that have each taken on a uniqueidentity; however, each of them contributes to the whole makeup and identity of the city.The same can be seen in other cities, such as Baltimore, which in a two-mile area has theInner Harbor, Little Italy, and Fell’s Point, with each area complementing the other zones.From Miami to San Francisco to New Orleans, all cities across America, and throughoutthe world for that matter, contain these identifiable natural areas.

Applying several other ecological principles, Park also noted that some areas (orspecies) may invade and dominate adjacent areas (species). The dominated area or speciescan recede, migrate to another location, or die off. In wildlife, an example is the incredi-ble proliferation of a weed called kudzu, which grows at an amazing pace and has verylarge leaves. It grows on top of other plants, trees, fields, and even houses, seeking to covereverything in its path and steal all of the sunlight needed by other plants. Introduced tothe United States in the 1800s at a world exposition, this weed was originally used to stoperosion but got out of control, especially in the southeastern region of the United States.Now the weed costs the government more than $350 million each year in destruction ofcrops and other flora. This is a good example of a species that invades, dominates, andcauses the recession of other species in the area.

A similar example can be found in the introduction of bison on Santa Catalina Island offthe Southern California coast in the 1930s. About three dozen buffalo were originallyimported to the island for a movie shoot, and the producers decided not to spend the moneyto remove them after the shoot, so they have remained and multiplied. Had this have occurredin many parts of the United States, it would not have caused a problem. However, the largestmammal native to the island before the bison was a four-pound fox. So the buffalo—nownumbering in the hundreds, to the point where several hundred were recently shipped to theirnative western habitat—have destroyed much of the environment, driving to extinction someplants and animals unique to Catalina Island. Like the kudzu, the bison came to dominate theenvironment; in this case, other species couldn’t move off the island and died off.

Park claimed that a similar process occurs in human cities as some areas invade otherzones or areas, and the previously dominant area must relocate or die off. This is easy tosee in modern times with the growth of what is known as urban sprawl. Geographers andurban planners have long acknowledged the detriment to traditionally stable residentialareas when businesses move into an area. Some of the most recent examples involve thebattles between long-time homeowners against the introduction of malls, businesses, andother industrial centers in a previously zoned residential district. The media have docu-mented such fights, especially with the proliferation of such establishments as Wal-Mart orSuper K-Marts, which residents perceive, and perhaps rightfully so, as an invasion. Such aninvasion can create chaos in a previously stable residential community due to increasedtraffic, transient population, and perhaps most important, crime. Furthermore, some cities

384 CRIMINOLOGICAL THEORY: A TEXT/READER

Section VI � The Chicago School and Cultural/Subcultural Theories of Crime 385

are granting power to such development through eminent domain, in which the local gov-ernment can take land from the homeowners to rezone and import businesses.

When Park developed his theory of ecology, he observed the trend of businesses andfactories invading the traditionally residential areas of Chicago, which caused majorchaos and breakdown in stability in those areas. Readers, especially those who were raisedin suburban or rural areas, can likely relate to this; going back to where they grew up, theycan often see fast growth. Such development can devastate the informal controls such asneighborhood networks or family ties, etc.) as a result of invasion by a highly transientgroup of consumers and residents who do not have strong ties to the area.

This leads to a psychological indifference toward the neighborhood, in which no onecares about protecting the community any longer. Those who can afford to leave the areado, and those who can’t afford to get out remain until they can save enough money to doso. When Park presented his theory of ecology in the 1920s, factories moving into theneighborhood often meant having a lot of smoke billowing out of chimneys. No onewanted to live in such a place, particularly at a time when the effects of pollution were notunderstood and such smoke stacks had no filters. Certain parts of Chicago and other U.S.cities were perpetually covered by smog these factories created. In highly industrial areas,the constant and vast coverage of smoke and pollutants made it seem to be snowing orovercast most of the time. It is easy to see how such invasions can completely disrupt thepreviously dominant and stable residential areas of a community.

Park’s ideas became even more valid and influential with the complementary perspec-tive offered by Ernest W. Burgess.5 He proposed a theory of city growth in which cities wereseen as growing not simply on the edges but from the inside outward. It is easy to observecities growing on the edges, as in the example of urban sprawl, but Burgess claimed that thesource of the growth was in the center of the city. Growth of the inner city puts pressure onadjacent zones, which in turn begin to grow into the next adjacent zones, following the eco-logical principle of “succession” identified by Park. This type of development is referred toas radial growth, meaning beginning on the inside and rippling outward.6

An example of this can be seen by watching a drop of water fall into the center of abucket filled with water. The waves from the impact will form circles that ripple outward.This is exactly how Burgess claimed that cities grow. Although the growth of cities is mostvisible on the edges, largely due to development of business and homes where only treesor barren land existed before, the reason for growth on the edges is due to pressure form-ing from the very heart of the city. Another good analogy is the “domino effect,” becausepressure from the center leads to pressure to grow on the next zone, which leads to pres-sure on the adjacent zones, and so forth.

Burgess also specified the primary zones—five pseudo-distinctive natural areas in aconstant state of flux due to growth—that all cities appear to have. He depicted these zonesas a set of concentric circles. The first innermost circle was called Zone I, or the centralbusiness district due to cost. This area of a city contains the large business buildings, mod-ern skyscrapers that are home to banking, chambers of commerce, courthouse, and otheressential business/political centers such as police headquarters, post office, and so on.

5Ernest W. Burgess, “The Growth of the City,” in The City, ed. Robert Park, Ernest W. Burgess, and Roderick D.McKenzie (Chicago: University of Chicago Press, 1928).

6See Vold et al., Theoretical Criminology, 118–21.

Just outside the business district is the unnumbered “factory zone.” It is perhaps themost significant in terms of causing crime because it invaded the previously stable resi-dential zones in Zone II, which was identified as the zone in transition. Zone II is appro-priately named because it was truly in a state of transition from residential to industrial,primarily because this was the area of the city in which business and factories were invad-ing residential areas. Zone II was the area that was most significantly subjected to the eco-logical principles Park suggested: invasion, domination, recession, and succession.Subsequent criminological theorists focused on this zone.

According to Burgess’s theory of concentric circles, Zone III was the “workingmen’shomes,” relatively modest houses and apartment buildings; Zone IV consisted of higher-priced family dwellings and more expensive apartments, and Zone V was the suburban orcommuter zone. These outer three zones Burgess identified were of less importance interms of crime, primarily because as a general rule, the further a family could move outof the city the better the neighborhood was in terms of social organization and the lowerthe rate of social ills (e.g., poverty, delinquency). The important point of this theory ofconcentric circles is that the growth of each inner zone puts pressure on the next zone togrow and push into the next adjacent zone.

It is easy for readers to see examples of concentric circles theory. Wherever you livein the United States, any major city provides real-life evidence of the validity of this per-spective. For example, whether people drive on Interstate 95 through Baltimore orInterstate 5 through Los Angeles, they will see the same pattern of city structure. As theyapproaches each of the cities, they see suburban wealth in the homes and buildings, oftenhidden by trees off the highway. Closer to the cities, they see homes and buildings deteri-orate in terms of value. Because parts of the highway systems near Baltimore and LosAngeles are somewhat elevated above the ground, drivers entering Zone II can easily seethe prevalence of factories and the highly deteriorated nature of the areas. Today, many20th-century factories have been abandoned or are limited in use; these factory zonesconsist of rusted-out or demolished buildings. Zone II is also often the location of subsi-dized or public housing. Only the people who can’t afford to live anywhere else are forcedto live in these neighborhoods. Finally, as drivers enter the inner city of skyscrapers, theconditions improve dramatically because the major businesses have invested their moneyhere. Compared to Zone II, this innermost area is a utopia.

This theory applies around the world, and we challenge readers to find any major citythroughout the world that did not develop this way. Nowadays, some attempts have beenmade to plan their community development, and others have experienced the conver-gence of several patterns of concentric circles as central business districts (i.e., Zone Is)are developed in what was previously suburb (i.e., Zone Vs). However, for the most part,the theoretical framework of concentric circles still has a great deal of support. In fact,even cities found in Eastern cultures have evolved this way. Therefore, Park’s applicationappears to be correct: Cities grow in a natural way across time and place, abiding by thenatural principles of ecology.

Shaw and McKay’s Theory of Social Disorganization

Clifford Shaw and Henry McKay drew heavily on their colleagues at the University ofChicago in devising their theory of social disorganization, which became known as the

386 CRIMINOLOGICAL THEORY: A TEXT/READER

Section VI � The Chicago School and Cultural/Subcultural Theories of Crime 387

Chicago School theory of criminology.7 Shaw had been producing excellent case studiesfor years on the individual (i.e., micro) level before he took on theorizing on the macro(i.e., structural) level of crime rates.8 However, once he began working with McKay, hedevised perhaps the most enduring and valid model of why certain neighborhoods havemore social problems, such as delinquency, than others.

In this model, Shaw and McKay proposed a framework that began with the assump-tion that certain neighborhoods in all cities have more crime than other parts of the city,most of them located in Burgess’s Zone II, which is the zone in the transition from resi-dential to industrial, due to the invasion of factories. According to Shaw and McKay, theneighborhoods that have the highest rates of crime typically have at least three commonproblems (see Figure 6.1): physical dilapidation, poverty, and heterogeneity (which is afancy way of saying a high cultural mix). There were other common characteristics tothese neighborhoods that Shaw and McKay noted, such as a highly transient population,meaning that people constantly move in and out of the area, as well as unemploymentamong the residents of the neighborhood.

As noted in Figure 6.1, other social ills are included as antecedent factors in thetheoretical model. The antecedent social ills tend to lead to a breakdown in socialorganization, which is why this model is referred to as the theory of social disorgani-zation. Specifically, it is predicted that the antecedent factors of poverty, heterogene-ity, and physical dilapidation lead to a state of social disorganization, which in turnleads to crime and delinquency. This means that the residents of a neighborhood that fits the profile of having a high rate of poor, culturally mixed residents in a dilap-idated area cannot come together to solve problems, such as delinquency amongyouth.

One of the most significant contributions of Shaw and McKay was that they demon-strated that the prevalence and frequency of various social ills—such as poverty, disease,and low birth weight—tend to overlap with higher delinquency rates. Regardless of whatsocial problem is measured, higher rates are almost always clustered in the zone in tran-sition. Shaw and McKay believed there was a breakdown of informal social controls inthese areas and that children began to learn offending norms from their interactions withpeers on the street, through what they called “play activities.”9 Thus, the breakdown in theconditions of the neighborhood leads to social disorganization, which in turn leads todelinquents learning criminal activities from older youth in the neighborhood.Ultimately, the failure of the neighborhood residents to organize themselves allows theolder youth to govern the behavior of the younger children. Basically, the older youth inthe area provide a system of organization where the neighborhood cannot, so youngerchildren follow them.

7Clifford Shaw and Henry D. McKay, Juvenile Delinquency and Urban Areas (Chicago: University of Chicago Press,1942); Clifford Shaw and Henry D. McKay, Juvenile Delinquency and Urban Areas, Rev. ed (Chicago: University ofChicago Press, 1969).

8Clifford Shaw, Brothers in Crime (Chicago: University of Chicago Press, 1938); Clifford Shaw, The Jackroller (Chicago:University of Chicago Press, 1930); Clifford Shaw, The Natural History of a Delinquent Career (Chicago: University ofChicago Press, 1931).

9Shaw, Brothers in Crime, 354–55.

One of the best things about Shaw and McKay’s theoretical model is that theresearchers support their propositions with data from the U.S. census and city recordsshowing that neighborhoods with high rates of poverty and physical dilapidation andhigh cultural mix also had the highest rates of delinquency and crime. Furthermore, thehigh rates of delinquency and other social problems were consistent with Burgess’sframework of concentric circles, in that the highest rates were observed for the areasthat were in Zone II, the zone in transition. There was an exception to the model: TheGold Coast area along the northern coast of Lake Michigan was notably absent from thehigh rates of social problems, particularly delinquency, even though it was geographi-cally in Zone II according to the otherwise consistent model of concentric circles andneighborhood zones.

So the findings of Shaw and McKay were as predicted in the sense that high delin-quency rates occurred in areas where factories were invading the residential district.Furthermore, Shaw and McKay’s longitudinal data showed that it did not matter which

388 CRIMINOLOGICAL THEORY: A TEXT/READER

Poverty

Heterogeneity(cultural mix)

Physicaldilapidation

High mobilityof residents

Other social ills(e.g., infant

mortality, highrate of disease)

Socialdisorganization

CriminalBehavior

Figure 6.1 Model of Shaw and McKay’s Theory of Social Disorganization

Section VI � The Chicago School and Cultural/Subcultural Theories of Crime 389

MAP OFCHICAGO

RA

TE

S F

OR

NO

RT

H H

ALF

1.7

1.7

2.6

5.1

7.9RATES FOR ENTIRE ZONE

9.81.8V

2.5IV

4.5III

6.7II I

11.9

RA

TE

S F

OR

SO

UT

H H

ALF

8.8

5.7

3.3

1.9

LAK

E M

ICH

IGA

N

LAKECALUNET

Figure 6.2 Zone Map of Male Delinquents in Chicago 1925–1933

ethnic groups lived in Zone II (zone in transition); all groups (with the exception ofAsians) that lived in that zone had high delinquency rates during their residency. On theother hand, once most of an ethnic group had moved out of Zone II, the delinquency rateamong its youth decreased significantly.

This finding rejects the notion of social Darwinism because it is clearly not the cul-ture that influences crime and delinquency but rather the criminogenic nature of the

environment. If ethnicity or race made a difference, the delinquency rates in Zone IIwould fluctuate based on who lived there, but the rates continued to be high from onegroup to the next. Rather, the zone determined the rates of delinquency.

Reaction and Research on Social Disorganization Theory

Over the last few decades, the Chicago School theoretical framework has received anenormous amount of attention from researchers.10 Virtually all of the research has sup-ported Shaw and McKay’s version of social disorganization and the resulting high crimerates in neighborhoods that exhibit such deprived conditions. Modern research has sup-ported the theoretical model proposed by Shaw and McKay, specifically in terms of thehigh crime rates in disorganized neighborhoods. Also, virtually every city that has an ele-vated highway (such as Richmond, Virginia; Baltimore, Maryland; Los Angeles,California), visually supports Shaw and McKay’s model of crime in concentric circles.Drivers entering those cities can see the pattern of dilapidated structures in the zone oftransition surrounding the inner-city area. Before and after this layer of dilapidated struc-tures, drivers encounter a layer of houses and residential areas that seem to increase inquality as the driver gets further away from the inner-city area.

Some critics, however, have raised some valid concerns regarding the original model,arguing that Shaw and McKay’s original research did not actually measure their primaryconstruct: social disorganization. Although this criticism is accurate, recent research hasshown that the model is valid even when valid measures of social disorganization areincluded in the model.11 Such measures of social disorganization include simply askingmembers of the neighborhood how many neighbors they know by name or how oftenthey observe unsupervised peer groups in the area.

Additional criticisms of Shaw and McKay’s formulation of social disorganizationfocus on the emphasis that the theory places on the macro, or aggregate, level ofanalysis. Although their theory does a good job of predicting which neighborhoodshave higher crime rates, the model does not even attempt to explain why most youthsin the worst areas do not become offenders. Furthermore, their model does notattempt to explain why some youths—although a very small number—in the bestneighborhoods (in Zone V) choose to commit crime. However, the previous case

390 CRIMINOLOGICAL THEORY: A TEXT/READER

10John Laub, “Urbanism, Race, and Crime,” Journal of Research in Crime and Delinquency 20 (1983): 283–98; RobertSampson, “Structural Sources of Variation in Race-Age-Specific Rate of Offending Across Major U.S. Cities,”Criminology 23 (1985): 647–73; J. L. Heitgard and Robert J. Bursik, “Extracommunity Dynamics and the Ecology ofDelinquency,” American Journal of Sociology 92 (1987): 775–87; Robert Bursik, “Social Disorganization and Theoriesof Crime and Delinquency: Problems and Prospects,” Criminology 26 (1988): 519–51; Ralph Taylor and JeanetteCovington, “Neighborhood Changes in Ecology and Violence,” Criminology 26 (1988): 553–89; Robert Bursik,“Ecological Stability and the Dynamics of Delinquency,” in Crime and Community, ed. Albert J. Reiss and Morris H.Tonry (Chicago: University of Chicago Press, 1986), 35–66; Robert Sampson, “Transcending Tradition: NewDirections in Community Research, Chicago Style—The American Society of Criminology 2001 Sutherland Address,”Criminology 40 (2002): 213–30; Robert Sampson, J. D. Morenoff, and T. Gannon-Rowley, “Assessing ‘NeighborhoodEffects’: Social Processes and New Directions in Research,” Annual Review of Sociology 28 (2002): 443–78; P. O.Wikstrom and Rolf Loeber, “Do Disadvantaged Neighborhoods Cause Well-Adjusted Children to Become AdolescentDelinquents?” Criminology 38 (2000): 1109–42.

11Robert Sampson and W. Byron Groves, “Community Structure and Crime: Testing Social Disorganization Theory,”American Journal of Sociology 94 (1989): 774–802.

Section VI � The Chicago School and Cultural/Subcultural Theories of Crime 391

studies completed and published by Clifford Shaw attempted to address the individ-ual (micro) level of offending.

Also, there was one notable exception to Shaw and McKay’s proposition that all ethnic/racial groups have high rates of delinquency/crime while they live in Zone II. Evidenceshowed that when Japanese made up a large portion of residents in this zone in transi-tion, they had very low rates of delinquency. Thus, as in most theoretical models in socialscience, there was an exception to the rule.

Perhaps the biggest criticism of Shaw and McKay’s theory, one that has yet to be ade-quately addressed, deals with their blatant neglect in proposing ways to ameliorate themost problematic source of criminality in the Zone II, transitional zone neighborhoods.Although they clearly point to the invasion of factories and businesses into residentialareas as a problem, they do not recommend how to slow such invasion. This is likely dueto political and financial concerns: The owners of the factories and businesses partiallyfinanced their research and later funded their primary policy implementation.Furthermore, this neglect is represented in their failure to explain the exception of theGold Coast in their results and conclusions.

Despite the criticisms and weaknesses of the Chicago School perspective of crimi-nology, this theory resulted in one of the largest programs to date in attempting to reducedelinquency rates. Clifford Shaw was put in charge of establishing the Chicago AreaProject (CAP), which created neighborhood centers in the most crime-ridden neighbor-hoods of Chicago. These centers offered activities for youth and tried to establish tiesbetween parents and officials in the neighborhood. Although this program was never sci-entifically evaluated, it still exists, and many cities have implemented programs that arebased on this model. For example, Boston implemented a very similar program, whichwas evaluated by Walter Miller.12 This evaluation showed that although the project waseffective in establishing relationships and interactions between local gangs and commu-nity groups and in providing more educational and vocational opportunities, it seemedto fail in reducing delinquent/criminal behavior. Thus, the overall conclusion made byexperts was that the Boston project and other similar programs, like the CAP, typically failto prevent criminal behavior.13

yy Cultural and Subcultural Theories of CrimeCultural/subcultural theories of crime assume that there are unique groups in societythat socialize their children to believe that certain activities that violate conventional laware good and positive ways to behave. Although it is rather difficult to find large groupsof people or classes that fit this definition, it may be that some subcultures or isolatedgroups of individuals buy into a different set of norms than the conventional, middle-class set of values.

12Walter B. Miller, “The Impact of a ‘Total-Community’ Delinquency Control Project,” Social Problems 10 (1962): 168–91.

13For a review, see Richard Lundman, Prevention and Control of Juvenile Delinquency, 2nd ed. (New York: OxfordUniversity Press, 1993). Also see review by Vold et al., Theoretical Criminology, 125–26.

Early Theoretical Developments andResearch in Cultural/Subcultural Theory

One of the key developments of cultural theory is the 1967 work of Ferracuti andWolfgang, who examined the violent themes of a group of inner-city youth fromPhiladelphia.14 Ferracuti and Wolfgang’s primary conclusion was that violence is a cul-turally learned adaptation to deal with negative life circumstances and that learning suchnorms occurs in an environment that emphasizes violence over other options.15 Theseresearchers based their conclusion on an analysis of data that showed great differences inrates of homicide across racial groups. However, Ferracuti and Wolfgang were clear thattheir theory was based on subcultural norms. Specifically, they proposed that no subcul-ture can be totally different from or totally in conflict with the society of which it is apart.16 This brings the distinction of culture and subculture to a forefront.

A culture represents a distinct set of norms and values among an identifiable groupof people, values that are summarily different from those of the mainstream culture. Forexample, communism is distinctly different from capitalism because it emphasizes equal-ity over competition, and it values utopia (i.e., everyone gets to share all profits) over theidea that the best performer gets the most reward. So it can be said that communists tendto have a different culture than capitalists. There is also a substantial difference between aculture and a subculture, which is typically only a pocket of individuals who may have aset of norms that deviate from conventional values. Therefore, what Ferracuti andWolfgang developed is not so much a cultural theory as much as a subcultural theory.

This is also seen in the most prominent (sub)culture theory, which was presented byWalter Miller.17 Miller’s theoretical model proposed that the entire lower class had its owncultural value system. In this model, virtually everyone in the lower class believed in andsocialized the values of six focal concerns: fate, autonomy, trouble, toughness, excitement,and smartness. Fate means luck, or whatever life dealt you; it disregards responsibility andaccountability for one’s actions. Autonomy is the value of independence from authority.Trouble means staying out of legal problems, as well as getting into and out of personal dif-ficulties (e.g., pregnancy). Toughness is maintaining your reputation on the street in manyways. Excitement is the engagement in activities, some illegal, that help liven up an other-wise mundane existence of being lower class. Smartness puts an emphasis on “street smarts”or the ability to con others. Miller thought that members of the lower class taught these sixfocal concerns as a culture or environment (or “milieu,” as stated in the title of his work).

A more recent subculture model, proposed by Elijah Anderson, has received a lot ofattention in the past few years.18 This theory focuses on African Americans; because of thevery deprived conditions in the inner cities, Black Americans feel a sense of hopelessness,isolation, and despair, Anderson asserts. He clearly notes that although many African

392 CRIMINOLOGICAL THEORY: A TEXT/READER

14Vold et al., Theoretical Criminology, 165–69; Frank Schmalleger, Criminology Today, 4th ed. (Upper Saddle River, NJ:Prentice Hall, 2006).15Franco Ferracuti and Marvin Wolfgang, The Subculture of Violence: Toward an Integrated Theory of Criminology(London: Tavistock, 1967).

16As quoted from Schmalleger, Criminology Today, 230–31.17Walter B. Miller, “Lower Class Culture as a Generating Milieu of Gang Delinquency,” Journal of Social Issues 14(1958): 5–19.18Elijah Anderson, Code of the Streets (New York: W. W. Norton, 1999).

Section VI � The Chicago School and Cultural/Subcultural Theories of Crime 393

Americans believe in middle-class values, these values have no weight on the street, par-ticularly among young urban males. According to Anderson, The Code of the Streets,which was the appropriate title of his book, is to maintain one’s reputation and demandrespect. For example, to be disrespected (“dissed”) is considered grounds for a physicalattack. Masculinity and control of one’s immediate environment are treasured character-istics; this is perceived as the only thing people can control, given the harsh conditions inwhich they live (e.g., unemployment, poverty, etc.).

Criticisms of Cultural Theories of Crime

Studies on cultural theories of crime, at least in the United States, find no largegroups that blatantly deny the middle-class norms of society. Miller’s model of lower-class focal concerns is not consistent across the entire lower class. Studies show that mostadults in the lower-class attempt to socialize their children to believe in conventional val-ues, such as respect for authority, hard work, and delayed gratification, and not the focalconcerns that Miller specified in his model.19 Ferracuti and Wolfgang admitted that theirresearch findings led them to conclude that their model was more of a subcultural per-spective and a distinctly different culture. There may be small groups or gangs that havesubcultural normative values, but that doesn’t constitute a completely separate culture insociety. Perhaps the best subculture theories are those presented by Cohen or Clowardand Ohlin (see Section V), in their variations of strain theory that emphasize the forma-tions of gangs among lower-class male youth. If there are subcultural groups in U.S. soci-ety, they seem to make up a very small percentage of the population, which somewhatnegates the cultural/subcultural perspective of criminality. But this type of perspectivemay be important regarding the criminality of some youth offenders.

yy Policy ImplicationsMany of the policy implications that are suggested by the theoretical models proposed inthis chapter are rather ironic. Regarding social disorganization, there is a paradox thatexists, in the sense that the very neighborhoods most desperately in need of becomingorganized to fight crime are the same inner-city ghetto areas where it is, by far, most dif-ficult to cultivate such organization (e.g., neighborhood watch or “Block Watch” groups).Rather, the neighborhoods that have high levels of organization tend to be those thatalready have very low levels of crime because the residents naturally “police” their neigh-bors’ well-being and property; they have a stake in the area remaining crime-free.Although there are some anecdotal examples of success of neighborhood watch programsin high-crime neighborhoods, most of the empirical evidence shows that this approach is“almost uniformly unsupportive” in its ability to reduce crime in such neighborhoods.20

Furthermore, many studies of these neighborhood watch programs actually find that thegroups actually increase the fear of crime in a number of neighborhoods, perhaps due tothe heightened awareness regarding the crime issues in such areas.21

19Vold et al., Theoretical Criminology.

20John Worrall, Crime Control in America: An Assessment of the Evidence (Boston: Allyn & Bacon, 2006), 95.

21Ibid.

Perhaps the most notable program that resulted from the Chicago School/social dis-organization model—the Chicago Area Project (CAP), and similar programs—have beendubbed failures in reducing crime rates among the participants. Still, there have beensome advances in trying to get residents of high-crime areas to become organized infighting crime. The more specific the goals regarding crime reduction—such as morecareful monitoring of high-level offenders (such as more intensive supervised probation)or better lighting in dark places—the more effective the implementation will be.22

Regarding cultural/subcultural programs, some promising intervention and outreachprograms that have been suggested by such models. Many programs attempt to build pro-social attitudes among high-risk youth, often young children. For example, a recent eval-uation showed that a program called Peace Builders, which focuses on children in earlygrades, was effective in producing gains in conflict resolution, development of pro-socialvalues, and reductions in aggression; a follow-up showed that these attributes were main-tained for a long period of time.23 Another recent anti-aggression training program forfoster-home boys showed positive effects in levels of empathy, self-efficacy, and attribu-tion style among boys who had exhibited early-onset aggression.24 Ultimately, there areeffective programs out there that promote pro-social norms and culture. More effortsshould be given to promoting such programs that will help negate the anti-social culturalnorms of individuals, especially among high-risk youth.

yy ConclusionIn this section, we examined theoretical perspectives proposing that the lack of socialorganization in broken down and dilapidated neighborhoods is unable to contain delin-quency and crime. Furthermore, we discussed how this model of crime was linked toprocesses derived from ecological principles. This type of approach has been testednumerous times, and virtually all studies show that the distribution of delinquents/crimeactivity is consistent with this model.

We then discussed the ability of cultural and subcultural theories to explain criminalactivity. Empirical evidence shows that cultural values make a contribution to criminalbehavior, but that the existence of an actual alternative culture in our society has not beenfound. However, some subcultural pockets, particularly regarding inner-city youth gangs,certainly exist and provide some validity for this perspective of crime. Furthermore, theChicago School perspective plays a role because these subcultural groups tend to be foundin zones of transition.

Finally, we examined some policy implications that are suggested by these theoreticalmodels. Regarding social disorganization, we noted that neighborhood crime-fighting

394 CRIMINOLOGICAL THEORY: A TEXT/READER

22Lundman, Prevention and Control; Sampson and Groves, “Community Structure”; Vold et al., Theoretical Criminology.

23D . J. Flannery, M. I. Singer, and K. L. Wester, “Violence, Coping, and Mental Health in a Community Sample ofAdolescence,” Violence and Victims 18 (2003): 403–18; For a review of this research, see Stephen G. Tibbetts, “Perinataland Developmental Determinants of Early Onset of Offending: A Biosocial Approach for Explaining the Two Peaksof Early Antisocial Behavior,” in The Development of Persistent Criminality, ed. J. Savage (New York: Oxford UniversityPress, 2009), 179–201.

24K. Weichold, “Evaluation of an Anti-aggressiveness Training with Antisocial Youth,” Gruppendynamik 35 (2004):83105; for a review, see Tibbetts, “Perinatal and Developmental.”

Section VI � The Chicago School and Cultural/Subcultural Theories of Crime 395

groups are hardest to establish in high-crime neighborhoods and easiest to build in thoseneighborhoods with an already low rate of crime. Nevertheless, there have been some suc-cesses. We also looked at intervention and outreach programs based on the cultural andsubcultural perspectives.

yy Section Summary� We examined how principles of ecology were applied to the study of how cities

grow, as well as to the study of crime, by researchers at University of Chicago,which became known as the Chicago (or Ecological) School of criminology.

� We reviewed the various zones of concentric circles theory, also a key contribu-tion of the Chicago School of criminology, and explored which zones are mostcrime prone.

� We examined why the findings from the Chicago School of criminology showedthat social Darwinism was not accurate in attributing varying crime rates to eth-nicity or race.

� We reviewed much of the empirical evidence regarding the theory of social dis-organization and examined the strengths and weaknesses of this theoreticalmodel.

� We discussed the cultural/subcultural model presented by Ferracuti andWolfgang, as well as the cultural model of inner-city urban youth presented byAnderson.

� We discussed Miller’s theory of lower-class culture, particularly its six focal concerns.� We reviewed the strengths and weaknesses of cultural and subcultural theories of

crime, based on empirical evidence.

K E Y T E R M S

Chicago School ofcriminology

Concentric circles theory

Cultural/subculturaltheories

Ecological School/perspective

Focal concerns

Natural areas

Social Darwinism

Social disorganization

Zone in transition

D I S C U S S I O N Q U E S T I O N S

1. Identify and discuss an example of the ecological principles of invasion, domination, and suc-cession among animals or plants that was not discussed in this introduction.

2. Can you see examples of the various zones that Shaw and McKay described in thetown/city where you live (or the one nearest you)? Try obtaining a map or sketching a plotof this city or town, and then draw the various concentric circles where you think the zonesare located.

3. What forms of organization and disorganization have you observed in your own neighbor-hood? Try to supply examples of both, if possible.

4. Can you provide modern-day examples of different cultures/subcultures in the United States? Whatregions, or parts of the country, would you say have different cultures that are more conducive tocrime?

5. Do you know people who believe most or all of Miller’s focal concerns? What is their social class? Whatare their other demographic features (age, gender, urban/rural, etc.)?

6. Do you know individuals who seem to fit either Ferracuti and Wolfgang’s cultural theory orAnderson’s model of inner-city youth’s street code? Why do you believe they fit such a model?

W E B R E S O U R C E S

Chicago School of Criminology

http://www.helium.com/items/865770-an-overview-of-the-chicago-school-theories-of-criminology

http://www.associatedcontent.com/article/771561/an_overview_of_the_chicago_school_theories.html

Subcultural Theories

http://www.umsl.edu/~keelr/200/subcult.html

R E A D I N G

In this selection, Clifford Shaw and Henry McKay present a theoretical model of vari-ous characteristics of neighborhoods that contribute to higher crime and delinquencyrates. Specifically, they examine physical, economic, and population factors that con-tribute to higher rates of delinquency in certain communities. Such observations of cer-tain neighborhoods provide the basis for the theory of social disorganization (alsoknown as the Chicago or Ecological School of criminology). While reading this selec-tion, readers are encouraged to think about the places they have lived or visited that fitthese characteristics; it is likely such neighborhoods have high crime rates. Because thistheory fits virtually all cities around the world, and because of their methodology, Shawand McKay are generally considered two of the most prominent criminologists of the20th century.

396 CRIMINOLOGICAL THEORY: A TEXT/READER

SOURCE: Clifford R. Shaw and Henry D. McKay, Juvenile Delinquency and Urban Areas, 2nd ed. (Chicago: University of ChicagoPress, 1969), 140–63. Copyright 1969 University of Chicago Press. Reprinted by permission of The University of ChicagoPress.

Section VI � Delinquency Rates and Community Characteristics 397

The question has been asked many times: “Whatis it, in modern city life that produces delin-quency?” Why do relatively large numbers ofboys from the inner urban areas appear in courtwith such striking regularity, year after year,regardless of changing population structure orthe ups and downs of the business cycle?[Elsewhere] a different series of male delinquentswere presented which closely parallel one anotherin geographical distribution although widelyseparated in time, and the close resemblance ofall these series to the distribution of truants andof adult criminals was shown. More over, manyother community characteristics—medianrentals, families on relief, infant mortality rates,and so on—reveal similar patterns of variationthroughout the city. The next step would be todetermine, if possible, the extent to which thesetwo sets of data are related. How consistently dothey vary together, if at all, and how high is thedegree of association?

Where high zero-order correlations arefound to exist uniformly between two vari-ables, with a small probable error, it is possibleand valid to consider either series as anapproximate index, or indicator, of the other.This holds true for any two variables which areknown to be associated or to vary concomi-tantly. The relationship, of course, may beeither direct or inverse. In neither case, how-ever, is there justification in assuming, on thisbasis alone, that the observed association is ofa cause-and-effect nature; it may be, rather,that both variables are similarly affected bysome third factor. Further analysis is needed.Controlled experimentation is often useful inestablishing the degree to which a change inone variable “causes” or brings about a corre-sponding change in the other. In the social

field, however, experimentation is difficult.Instead, it is often necessary to rely uponrefined statistical techniques, such as partialcorrelation, which, for certain types of data,enable the investigator to measure the effectsof one factor while holding others relativelyconstant. By the method of successive redistri-bution, also, the influence of one or more vari-ables may be held constant. Thus, it is possibleto study the relationship between rates ofdelinquents and economic status for a singlenationality group throughout the city or forvarious nationality groups in the same area orclass of areas. This process may be extendedindefinitely, subject only to the limitations ofthe available data. In the analysis to be pre-sented, both of the latter methods have beenused in an attempt to determine how muchweight should be given to various more or lessinfluential factors.

Several practical considerations preventthe neat and precise statistical analysis whichwould be desirable. The characteristics studiedrepresent only a sampling of the myriad formsin which community life and social relation-ships find expression. The rate of delinquentsmust itself be thought of as an imperfect enu-meration of the delinquents and an index ofthe larger number of boys engaging in offi-cially proscribed activities. Not only will therebe chance fluctuations in the amount ofalleged delinquency from year to year, but thepolicy of the local police officer in referringboys to the Juvenile Court, the focusing of thepublic eye upon conditions in an area, andnumerous other matters may bring about achange in the index without any essential changein the underlying delinquency-producinginfluences in the community or in the behavior

Delinquency Rates and Community Characteristics

Clifford R. Shaw and Henry D. McKay

resulting therefrom. If the infant mortalityrates or the rates of families on relief arelooked upon as indexes of economic status orof the social organization of a community, itis obvious that they can be considered onlyvery crude indicators at best. The perturbinginfluence of other variables must always beconsidered.

Certain exceptional conditions are knownto limit the value of other variables chosen asindicators of local community differentiation.Median rental has been used widely because ofits popularity as an index of economic status,although in Chicago such an index is far fromsatisfactory when applied to areas of coloredpopulation. The Negro is forced to pay consider-ably higher rents than the whites for comparablehousing—thus his economic level is made toappear higher on the basis of rental than it actu-ally is. Similarly, rates of increase or decrease ofpopulation are modified in Negro areas byrestrictions on free movement placed upon theNegro population. Thus, in certain areas thepopulation is increasing where it normallywould be expected to decrease if there were nosuch barriers. Likewise, the percentage of fami-lies owning homes is not entirely satisfactory asan economic index in large urban centers, wheremany of the well-to-do rent expensive apart-ments. It is, however, an indication of the relativestability of population in an area.

Correlation of series of rates based on geo-graphical areas is further complicated by thefact that magnitude of the coefficient is influ-enced by the size of the area selected. This ten-dency has been noted by several writers, but nosatisfactory solution of the problem has beenoffered. If it be borne in mind that a correla-tion of area data is an index of geographicalassociation for a particular type of spatial divi-sion only, rather than a fixed measure of func-tional relationship, it will be apparent that achange in area size changes the meaning of thecorrelation. Thus, an r of .90 or above for twoseries of rates calculated by square-mile areas

indicates a high degree of association betweenthe magnitudes of the two rates in most of thesquare miles but does not tell us the exactdegree of covariance for smaller or larger areas.

With these limitations clearly in mind, anumber of correlation coefficients and tables ofcovariance are presented. The statistical datacharacterizing and differentiating local urbanareas may be grouped under three headings: (1) physical status, (2) economic status, and (3) population composition. These will beconsidered, in turn, in relation to rates ofdelinquents.

yy Indexes of PhysicalStatus in Relation toRates of Delinquents

The location of major industrial and commer-cial developments, the distribution of buildingscondemned for demolition or repair, and thepercentage increase or decrease in populationby square-mile areas were presented [elsewhere]as indications of the physical differentiation ofareas within the city. Quantitative measures ofthe first two are not available, but inspection ofthe distribution maps shows clearly that thehighest rates of delinquents are most frequentlyfound in, or adjacent to, areas of heavy industryand commerce. These same neighborhoodshave the largest number of condemned build-ings. The only notable exception to this general-ization, for Chicago, appears in some of theareas south of the central business district.

There is, of course, little reason to postu-late a direct relationship between living inproximity to industrial developments andbecoming delinquent. While railroads andindustrial properties may offer a field for delin-quent behavior, they can hardly be regarded asa cause of such activities. Industrial invasionand physical deterioration do, however, makean area less desirable for residential purposes.As a consequence, in time there is found a

398 CRIMINOLOGICAL THEORY: A TEXT/READER

Section VI � Delinquency Rates and Community Characteristics 399

movement from this area of those people ableto afford more attractive surroundings.Further, the decrease in the number of build-ings available for residential purposes leads to adecrease in the population of the area.

Population Increase or Decrease. Increase ordecrease of population and rates of delin-quents, by square-mile areas, do not exhibit alinear relationship. A relatively slight differ-ence in rate of decrease of population, or ofrate of increase for areas where the increase isslight, is generally associated with a consider-able change in rates of delinquents; while forlarge differences in rates of increase of popula-tion, where increase is great, there is little or noconsistent difference in rates of delinquents.Thus, areas increasing more than 70 per centshow no corresponding drop in rates of delin-quents, although the relationship is clear up tothis point. . . .

[T]here is a similarity between the patternof distribution of delinquency and that ofpopulation growth or decline. The data do notestablish a causal relationship between the twovariables, however. The fact that the popula-tion of an area is decreasing does not impel aboy to become delinquent. It may be said,however, that decreasing population is usuallyrelated to industrial invasion of an area andcontributes to the development of a generalsituation conducive to delinquency.

yy Population Compositionin Relation to Rates ofDelinquency

In Chicago, as in other northern industrialcities, as has been said, it is the most recentarrivals—persons of foreign birth and thosewho have migrated from other sections of thiscountry—who find it necessary to make theirhomes in neighborhoods of low economic level.Thus the newer European immigrants are

found concentrated in certain areas, whileNegroes from the rural South and Mexicansoccupy others of comparable status. Neither ofthese population categories, considered sepa-rately, however, is suitable for correlation withrates of delinquents, since some areas of highrates have a predominantly immigrant popula-tion and others are entirely or largely Negro.Both categories, however, refer to groups of loweconomic status, making their adjustment to acomplex urban environment. Foreign-born andNegro heads of families will therefore be con-sidered together in order to study this segrega-tion of the newer arrivals, on a city-wide scale.

Percentage of Foreign-Born and Negro Heads ofFamilies.1 When the rates of delinquents in the1927–33 series are correlated with the percent-age of foreign-born and Negro heads of familiesas of 1930, by 140 square-mile areas, the coeffi-cient is found to be .60 ± .03. Similarly, when the1917–23 delinquency data are correlated withpercentages of foreign-born and Negro heads offamilies for 1920, by the 113 areas into whichthe city was divided for that series, the coeffi-cient is .58 ± .04. When rates of delinquents arecalculated for the classes of areas . . . wide vari-ations are found between the rates in the classeswhere the percentage of foreign-born andNegro heads of families is high and in thosewhere it is low. . . . Since the number of foreign-born heads of families in the populationdecreased and the number of Negroes increasedbetween 1920 and 1930, the total proportions offoreign-born and Negro heads of families ineach class do not correspond. The variationwith rates of delinquents, however, remainsunchanged.

While it is apparent from these data thatthe foreign born and the Negroes are concen-trated in the areas of high rates of delinquents,the meaning of this association is not easilydetermined. One might be led to assume thatthe relatively large number of boys broughtinto court is due to the presence of certain

racial or national groups, were it not for thefact that the population composition of manyof these neighborhoods has changed com-pletely, without appreciable change in theirrank as to rates of delinquents. Clearly, one mustbeware of attaching causal significance to race ornativity. For, in the present social and economicsystem, it is the Negroes and the foreign born, orat least the newest immigrants, who have leastaccess to the necessities of life and who are there-fore least prepared for the competitive struggle.It is they who are forced to live in the worst slumareas and who are least able to organize againstthe effects of such living.

In Chicago three kinds of data are availablefor the study of nativity, nationality, and race inrelation to rates of delinquents. These data con-cern (1) the succession of nationality groups inthe high-rate areas over a period of years; (2)changes in the national and racial backgroundsof children appearing in the Juvenile Court; and(3) rates of delinquents for particular racial,nativity, or nationality groups in different typesof areas at any given moment. In evaluating thesignificance of community characteristicsfound to be associated with high rates of delin-quents, the relative weight of race, nativity, andnationality must be understood. . . .

It appears to be established, then, that eachracial, nativity, and nationality group inChicago displays widely varying rates of delin-quents; that rates for immigrant groups in par-ticular show a wide historical fluctuation; thatdiverse racial, nativity, and national groupspossess relatively similar rates of delinquentsin similar social areas; and that each of thesegroups displays the effect of disproportionateconcentration in its respective areas at a giventime. In the face of these facts it is difficult tosustain the contention that, by themselves, thefactors of race, nativity, and nationality arevitally related to the problem of juvenile delin-quency. It seems necessary to conclude, rather,that the significantly higher rates of delinquents

found among the children of Negroes, the foreign born, and more recent immigrants areclosely related to existing differences in theirrespective patterns of geographical distribu-tion within the city. If these groups were foundin the same proportion in all local areas, exist-ing differences in the relative number of boysbrought into court from the various groupsmight be expected to be greatly reduced or todisappear entirely.

It may be that the correlation betweenrates of delinquents and foreign-born andNegro heads of families is incidental to rela-tionships between rates of delinquents andapparently more basic social and economiccharacteristics of local communities. Evidencethat this is the case is seen in two partial corre-lation coefficients computed. Selecting therelief rate as a fair measure of economic level,the problem is to determine the relative weightof this and other factors. The partial correla-tion coefficient between rate of delinquentsand percentage of families on relief, holdingconstant the percentage of foreign-born andNegro heads of families, in the 140 areas, is .76 ± .02. However, the coefficient for rates ofdelinquents and percentage of foreign-bornand Negro heads of families, when percentageof families on relief is held constant, is only .26± .05. It is clear from these coefficients, there-fore, that the percentage of families on relief isrelated to rates of delinquents in a more signif-icant way than is the percentage of foreign-born and Negro heads of families.

It should be emphasized that the highdegree of association between rates of delin-quents and other community characteris-tics . . . does not mean that these characteristicsmust be regarded as causes of delinquency, orvice versa. Within certain types of areas differ-entiated in city growth, these phenomenaappear together with such regularity that theirrates are highly correlated. Yet the nature of therelationship between types of conduct and

400 CRIMINOLOGICAL THEORY: A TEXT/READER

Section VI � Delinquency Rates and Community Characteristics 401

given physical, economic, or demographiccharacteristics is not revealed by the magnitudeeither of zero-order or partial correlation coef-ficients, or of other measures of association.

A high degree of association may lead tothe uncritical assumption that certain fac-tors are causally related, whereas furtheranalysis shows the existing association to beentirely adventitious. This is apparently thecase with the data on nativity, nationality, andrace. . . . That, on the whole, the proportion offoreign-born and Negro population is higherin areas with high rates of delinquents therecan be little doubt; but the facts furnish amplebasis for the further conclusion that the boysbrought into court are not delinquent becausetheir parents are foreign born or Negro butrather because of other aspects of the total sit-uation in which they live. In the same way, therelationship between rates of delinquents andeach associated variable should be explored,either by further analysis, by experimentation,or by the study of negative cases.

yy SummaryIt has been shown that, when rates of delin-quents are calculated for classes of areas

grouped according to rate of any one of anumber of community characteristics stud-ied, a distinct pattern appears—the two setsof rates in each case varying together. Whenvalues of these other community character-istics, in turn, are calculated for classes ofareas grouped by rate of delinquents, thesame consistent trends appear. . . . Thedata . . . indicate a high degree of associationbetween rates of delinquents and other com-munity characteristics when correlations arecomputed on the basis of values in square-mile areas or similar subdivisions, and a stillcloser general association by large zones orclasses of areas. . . .

yy Note1. The categories “foreign born” and “Negro”

are not compatible, since the former group is madeup primarily of adults, while the latter includes allmembers of the race. The classification “heads offamilies” has been used, therefore, foreign-born and Negro family heads being entirely comparablegroupings. The census classification “other races”has been included—a relatively small group, comprising Mexicans, Japanese, Chinese, Filipinos,etc.

R E V I E W Q U E S T I O N S

1. What do Shaw and McKay have to say about delinquency rates in neighborhoods located in or nearheavy industrial areas? What types of physical indicators are present in such areas?

2. What do Shaw and McKay conclude regarding the crime rates of neighborhoods containing a popula-tion composition high in minorities or recent immigrants?

3. What do Shaw and McKay claim would happen to delinquency/crime rates if racial/ethnic, as wellas recent immigrants, were equally distributed across all neighborhoods of a city (such asChicago)?

Although Shaw and McKay’s work linkingsocial disorganization to community crimerates exerted considerable influence on crimi-nological theory during the 1950s, by the mid-1980s, their perspective came to be consideredby many as “little more than an interestingfootnote in the history of community-relatedresearch” (Bursik 1986:36; see also, Pfohl1985:158). Since that time, however, the socialdisorganization perspective has experienced a

dramatic revitalization, reemerging from thedustbin of spent criminological paradigms tochallenge for the status as a preeminentmacro-level theory.

No single work did more to polish the tar-nished image of social disorganization theorythan Sampson and Groves’s essay, published in1989, “Community Structure and Crime:Testing Social-Disorganization Theory.” Theirresearch ostensibly showed that consistent

402 CRIMINOLOGICAL THEORY: A TEXT/READER

R E A D I N G

In this selection, Christopher Lowenkamp, Frank Cullen, and Travis Pratt present an empir-ical study that seeks to replicate the findings of a well-respected previous test of the ChicagoSchool model of social disorganization theory (as originally presented by Clifford Shaw andHenry McKay). Specifically, this well-respected previous test of this theoretical model wasperformed by Robert Sampson and W. Byron Grove (1989), using data from the 1982British Crime Survey. It generally supported the social disorganization theoretical frame-work, especially regarding the mediating effects of informal social controls in explaining theimpact of structural factors (e.g., poverty) on neighborhood crime rates. In this study,Lowenkamp et al. find that a replication of this earlier study by Sampson and Grove (1989),which utilizes more recently collected data, also revealed a high level of empirical supportfor the social disorganization theoretical framework. The few differences between this andthe earlier test (by Sampson and Grove) actually revealed more support for the ChicagoSchool model. While reviewing this study, readers are encouraged to consider the places theyhave lived or visited that fit these characteristics.

Replicating Sampson and Groves’s Test ofSocial Disorganization Theory

Revisiting a Criminological Classic

Christopher T. Lowenkamp, Francis T. Cullen, and Travis C. Pratt

SOURCE: Adapted from Christopher T. Lowenkamp, Francis T. Cullen, and Travis C. Pratt, “Replicating Sampson and Grove’sTest of Social Disorganization Theory: Revisiting a Criminological Classic.” Journal of Research in Crime and Delinquency 40,no. 4 (2003): 351–73. Copyright 2003 Sage Publications, Inc. Used by permission of Sage Publications, Inc.

Section VI � Replicating Sampson and Grove’s Test of Social Disorganization Theory 403

with the core predictions of the social disorga-nization perspective, the impact of structuralfactors on crime rates was mediated in impor-tant ways by direct measures of informal socialcontrols (Taylor 2001). It is noteworthy thatcontemporary publications now cite Sampsonand Groves’s (1989) work as providing “a con-vincing test of Shaw and McKay’s social disor-ganization thesis” (Bellair 1997:679) and as“one of the more important studies in crimi-nological literature over the past decade”(Veysey and Messner 1999: 156). It is amongthe most cited criminological works and hasbecome a staple in criminological textbookspublished after 1989 (see, e.g., Akers 1999;Siegel 2000; Sykes and Cullen 1992; Tittle andPaternoster 2000).1 In short, “CommunityStructure and Crime: Testing Social-Disorga -nization Theory” has emerged as a classic inthe field of criminology.

As with other classic empirical works,however, the question arises as to whetherSampson and Groves’s (1989) research offersvirtually unshakable support for social disor-ganization theory or merely produced an idio-syncratic finding that was unique to a certaintime and place. To gain insight on this issue,subsequent studies ideally would have repli-cated Sampson and Groves’s work in differentcontexts and with refined measures of thestudy’s core variables. But precise replicationsof their empirical test have been hampered bythe failure of existing macro-level data sets tocontain intervening, direct measures of infor-mal social controls, including the full set ofvariables specified in the original study. A lim-ited number of studies have moved beyondstructural proxies to assess components ofsocial disorganization or similar processes(Bellair 1997; Morenoff et al. 2001; Roundtreeand Land 1996; Roundtree and Warner 1999;Sampson et al. 1997; Sampson et al. 1999; Velez2001; Warner and Roundtree 1997). Yet, again,none has attempted to provide a systematic testof Sampson and Groves’s complete model.Accordingly, we have a classic study—cited

repeatedly in the literature—that remains, ineffect, nonreplicated.

In this context, we suggest an alternativestrategy for replicating Sampson and Groves’s(1989) apparent empirical confirmation ofsocial disorganization theory. We propose torevisit the British Crime Survey (BCS) (HomeOffice Crime and Criminal Justice Unit 1994)a decade later to investigate whether the find-ings reported in the original analysis willremain stable as social disorganization theorywould predict. If the findings change in mean-ingful ways, it would suggest that the credenceaccorded Sampson and Groves’s “classic” testis not warranted and that social disorganiza-tion theory cannot account for crime rates ina later time period, even though virtually thesame data set and measures are being used.Phrased differently, it would call into questionwhether the social disorganization perspectiveis a “general theory” capable of accounting forecological variations in crime rates acrosstime and place.

If, however, their findings remain largelyconstant, it would suggest that the initialanalysis by Sampson and Groves (1989) wasnot idiosyncratic but rather captured an ongo-ing empirical reality. Much as in other scien-tific fields, it would show that their conclusionscould be reproduced under “laboratory con-ditions” (so to speak) that were similar to, butindependent of, the original investigation.Admittedly, these findings would not necessar-ily mean that results from Britain would gen-eralize, without specification, to the UnitedStates and other social contexts. Nonetheless,these results would bolster social disorganiza-tion theory’s claim to be a perspective thatwarrants its place as a vital macro-level theoryof crime and as deserving of further empiricalinvestigation.

We begin by revisiting Sampson andGroves’s (1989) classic test in more detail. Thisdiscussion serves as a prelude for a replication oftheir study using 1994 data from the BCS. Thisyear of the BCS was chosen because it allows for

the closest replication of Sampson and Groves’sstudy. As we shall see, in important ways, theirfindings are largely replicated by our analysis,suggesting that social disorganization theory ispotentially useful in accounting for variation inmacro-level crime rates across communities.

Finally, we note the growing recognitionwithin criminology—and social science gener-ally—of the value of systematic replication(Gendreau in press). Breaking new theoreticaland/or empirical ground is perhaps of primeimportance, for such efforts potentially pushthe field to new horizons. Nevertheless, theemphasis on innovation over systematic repli-cation has a cost: the inability to build cumu-lative knowledge and to be able to state withconfidence what is, and is not, known. Evenwhen theories have been subjected to tens ofempirical tests, it is difficult to assess the per-spectives’ merits because of the failure of theresearch to build in an organized way on oneanother (e.g., measure all core constructs.include similar measures) (Kempf 1993). Inthis regard, the current study should be seen asan initial, but much needed, attempt to con-tribute to the cumulative knowledge base onSampson and Groves’s (1989) analysis of socialdisorganization theory.

yy Sampson and Groves’s Testof Social DisorganizationTheory

Measuring Social Disorganization:Intervening Variables

As Sampson and Groves (1989) noted,most ecological research inspired by Shaw andMcKay (1972) examined the effects of struc-tural antecedents (heterogeneity, socioeco-nomic status [SES], and mobility) on crimerates (see also Bursik 1986, 1988; Byrne andSampson 1986). Although suggestive, thesetheoretical tests failed to provide direct mea-sures of whether social disorganization

prevailed within the ecological units underinvestigation (Veysey and Messner 1999). Inessence, the lack of measures of the theorizedmediators/intervening variables have created a“black box” phenomenon in which little isknown about what exists between structuralvariables on the left side of a causal model andcrimes rates on the other side of the model.Because there are no measures of the interven-ing variables, assumptions and inferences—plausible as they may be—must be invoked toexplain the processes that might specify therelationships between the structural anddependent variables. Notably, this dearth ofattention to measures of mediating processesplagues not only social disorganization theory,but also ecological theories in general (Byrneand Sampson 1986).

To test social disorganization theory withdirect measures of intervening variables,Sampson and Groves (1989) used data fromthe 1982 BCS.2 Their goal was to examine themediating effects of intervening variables onthe structural antecedents hypothesized to becausal influences on social disorganizationand, subsequently, criminality within a com-munity. Building on Shaw and McKay’s (1972)original model of social disorganization,Sampson and Groves (1989) included threemeasures of community-level variables thoughtto cause social disorganization: low economicstatus, ethnic heterogeneity, residential mobil-ity. In extending Shaw and McKay’s (1972)model, Sampson and Groves specified vari-ables for two additional sources of social disor-ganization: urbanization and family disruption(a variable drawn from Sampson’s [1987] earlier work).

The uniqueness of Sampson and Groves’s(1989) work stems from the authors’ con-struction of direct measures of social disorga-nization. These were treated as interveningvariables in their main analyses. These mea-sures include sparse local friendship net-works, low organizational participation, andunsupervised teenage peer groups. These

404 CRIMINOLOGICAL THEORY: A TEXT/READER

Section VI � Replicating Sampson and Grove’s Test of Social Disorganization Theory 405

measures were largely developed based onworks by Kasarda and Janowitz (1974), Krohn(1986) and on the relation of these works toShaw and McKay’s (1972) theoretical frame-work. Each measure was based on aggregateddata from individual respondents in 238communities.

Empirical Support forSocial Disorganization Theory

To determine the impact of structuralantecedents on social disorganization and oncrime, Sampson and Groves (1989) developedand analyzed several weighted least squares(WLS) regression models. The results of theiranalyses indicated that each of the structuralfactors significantly predicted the degree towhich respondents perceived unsupervisedpeer groups to be a problem in their commu-nity (R2=.30). The model predicting the levelof local friendship networks indicated thatonly residential stability and urbanizationhad significant relationships with the depen-dent measure (R2 = .26). Only one structuralvariable, SES, had a significant relationshipwith the level of organizational participation(R2 = .07).

Turning to the results regarding thedependent variables, crime and delinquency,eight models were presented by Sampson andGroves (1989). Models predicting rates ofmugging (R2 = .61), stranger violence (R2 =.15), total victimization (R2 = .42), burglary(R2 = .39), auto theft (R2 = .30), and vandalism(R2 = .21) indicated, with few exceptions, thatthe intervening variables thought to measuresocial disorganization had significant relation-ships with the dependent variables and medi-ated some of the direct effects of the structuralvariables. Regarding rates of self-reportedoffending, unsupervised peer groups mediatedmuch of the effects of the structural variableswhen predicting violent offending (R2 = .06)Along with local friendship networks, unsu-pervised peer groups also mediated the effects

of the structural variables when predictingself-reported property offenses (R2 = .09).

Replicating Sampsonand Groves’s Test

Although Sampson and Groves’s (1989)study has not been systematically replicatedoutside their article,3 their original data haverecently been reassessed by Veysey andMessner (1999) through the use of LISRELinstead of WLS regression analysis. The find-ings from the reanalysis were largely consistentwith those from Sampson and Groves (1989):(1) the social disorganization variables medi-ated, although not completely, the effect ofstructural characteristics on rates of commu-nity victimization, and (2) the variable ofunsupervised peer groups had the largestimpact on the outcome measures. We shouldalso note, however, that Veysey and Messner’s(1999) LISREL analysis revealed that the mea-sures of structural antecedents had variousdirect effects on rates of crime (as measured byvictimization data). This finding, they suggest,supports a position that is conceptually incon-sistent with Sampson and Groves’s (1989) ver-sion of social disorganization theory becausethe disorganization perspective would arguethat the effects of structural factors should becompletely mediated by measures of eachcommunity’s level of disorganization.

Nevertheless, in their article, Sampsonand Groves (1989) did not contend that theintervening variables in their model mediatedfully the effects of the five structural factors.Their original path model showed directeffects of structural factors, although, admit-tedly, they highlighted the ability of their mea-sures of neighborhood disorganization andinformal social control to mediate the impactof these factors on levels of crime. Thus,Veysey and Messner’s (1999) findings that thesocial disorganization variables did not com-pletely mediate the relationships betweenthe structural variables and rates of crime is

perhaps best seen as an important clarifica-tion of, rather than a failure to replicate,Sampson and Groves’s (1989) analysis. In anycase, we also replicate Veysey and Messner’s(1999) analysis and discuss its implications forsocial disorganization theory.

yy MethodAs with many surveys conducted repeatedtimes, additions and deletions have been madeto the content of information gathered in theBCS. After reviewing the codebooks for theBCSs conducted after 1982, we determinedthat the data from the 1994 survey containthose questions and information most similarto the data used by Sampson and Groves(1989). More recent BCSs no longer containthe same or, in some cases, similar questionsemployed by Sampson and Groves in the con-struction of their measures. In any event, the1994 survey provided us with the data—collected a decade after the original study—toconstruct, in most instances, measures identi-cal to those used by Sampson and Groves. Ininstances where identical measures could notbe developed, very similar measures were constructed—each of which is noted below.The use of similar measures is significant becauseit means that differences in measurement can-not be employed to explain any divergent find-ings that might arise between the two studies.

Sample

The data for this study come from the 1994BCS. The sampling method used is a multistagerandom probability design where 600 postalcode sectors are selected by a systematic sam-pling method from a stratified list of postal codesectors. The probability for selection of any par-ticular postal code sector was proportional tothe total number of delivery points within thatsector. Once 600 sectors were selected, 36addresses were randomly selected from each

sector yielding a total of 21,600 addresses. Afterremoving all ineligible addresses and nonre-sponding households, the sample size wasreduced to 14,617. This averages approximately25 respondents per sector. The BCS collects dataon the characteristics of adults and children thatreside within the household as well as informa-tion pertaining to residential history, percep-tions of crime, problems in the area, socialhabits, type of community, community involve-ment, social cohesion, criminal victimization,and experience with the police.

This sampling method differs from thatused in 1982, where 60 addresses from 238electoral wards were selected. After removingineligible addresses and nonresponding house-holds, the final sample size for 1982 was10,905. Checks on the differences betweenthese two sampling frames indicate no appre-ciable differences in crime rate trends due tothe sampling frame (Lynn 1997).

Measures of Intervening Variables

The intervening variables in this modelare very similar to those used by Sampson andGroves (1989). We have included measures ofunsupervised teenage peer groups, localfriendship networks, and organizational par-ticipation. Sampson and Groves measuredlocal friendship networks based on responses tothe question that asked the number of friendsthat lived in the local community. A mean wascalculated for these responses and served as themeasure of local friendship networks. Ourlocal friendship indicator was derived fromresponses to the question, “Thinking of thepeople who live in this area, by this area, Imean a 10–15 minute walk from here, howmany would you regard as friends or acquain-tances?” We also calculated a mean responseand used this as our measure.

As measured by Sampson and Groves(1989), organizational participation was thepercentage of respondents that reported inmeetings of clubs or committees the week

406 CRIMINOLOGICAL THEORY: A TEXT/READER

Section VI � Replicating Sampson and Grove’s Test of Social Disorganization Theory 407

before the interview. Our measure of organiza-tional participation is the percentage of peoplethat reported participating in a meeting of aclub or committee the last time they were outin the evening.

The final intervening variable, unsuper-vised peer groups, was measured by Sampsonand Groves (1989) as the percentage ofrespondents that reported groups of teenagersmaking a nuisance of themselves as a verycommon problem in their neighborhood. Ourmeasure differed slightly in that the questionrelating to teenage groups asked respondentshow big a problem teenage groups hangingaround on the streets was in their neighbor-hood. We calculated the percentage of peoplethat reported teenage to be a very big problem.Although this measure differs from the 1982BCS, for the purpose at hand, it is most likely apreferable measure. As Sampson and Groves(1989) note, by removing the mention of “nui-sance” behavior, the concern that respondentsare confounding crime rates with “nuisance”behavior of teens is obviated.

Measures of Structural Variables

Consistent with Sampson and Groves(1989), we constructed measures of SES, resi-dential stability, ethnic heterogeneity, familydisruption, and urbanization. We followedidentical processes in constructing these mea-sures (e.g., formulas, summed z scores percent-ages, and means). Wherever possible, we usedthe same questions and responses.

SES is a composite of measure of z scores tap-ping education (the percentage of people witha higher degree or teaching certification),occupation (percentage in professional ormanagerial occupations using the BCS socialclass ratings), and income (percentage report-ing incomes in the top two income brackets).

Residential stability, as measured by Sampsonand Groves (1989), was the percentage of people

that were brought up in the area within a15-minute walk from home. Due to changes inthe questions on the BCS main questionnaire,our measure of residential stability is the per-centage of people that reported living in the area(within a 15 minute walk) for 10 or more years.

Ethnic heterogeneity is calculated using thesame formula as Sampson and Groves (1989)(1 – ∑pi2) where pi is the fraction of people ina particular group. Our measure of ethnic het-erogeneity differed from Sampson andGroves’s in that the 1994 BCS used sevenrather than five categories of ethnicity.

Family disruption was the sum of z scores forthe percentage of people surveyed that wereeither separated or divorced and the percent-age of people that were separated, divorced, orsingle and had children in their household.This measure is identical to that used bySampson and Groves (1989).

Urbanization, finally, was coded as either 0 or 1according to the BCS data, where 1 representsan urban area. In 1982, the Planning andResearch Applications Group’s (PRAG) inner-city designations were used to determinewhich areas were inner-city areas. In 1984, themethod for identifying inner-city areas waschanged to where inner-city status is deter-mined by the population density, percentage ofowner occupied housing, and/or the socialclass of the areas based on standard occupa-tional classification. This is the same proce-dure used in subsequent surveys.

Measure of Crime

Consistent with Sampson and Groves’s(1989) final analysis (as well as the reanalysisof the 1982 BCS data by Veysey and Messner1999), our measure of crime is the total victim-ization rate, which reflects the rate of personaland household victimization. This variableconsists of the sum of crimes that can be clas-

sified as personal or household victimizationand then standardizing it by the number ofrespondents in the area.

yy ResultsThe results of the replication of Sampson andGroves’s (1989) analysis using the 1994 BCSdata are presented here in two sections. First,we report the results of the social disorganiza-tion models using the 1994 BCS data to determine—on their own—whether they sup-port the major propositions specified by socialdisorganization theory according to Sampsonand Groves. Second, we present a series of statis-tical comparisons between the standardizedparameter estimates from the 1982 wave of theBCS to determine whether any of the resultsfrom the present replication study are signifi-cantly different from those of the original study.

Social DisorganizationModels From the 1994 BCS

As with Sampson and Groves’s (1989)analysis, our replication using the 1994 BCSdata proceeded in two stages. First, we examined

the effect of the five structural characteristicson the three measures of social disorganizationoutlined by Sampson and Groves. Second, theeffects of the social disorganization measuresas “intervening” variables (i.e., as mediators ofthe effects of the structural characteristics)predicting neighborhood crime rates wereassessed.

Structural Characteristicsand Social Disorganization

Table 1 contains the results of the WLSregression models testing the effects of thestructural characteristics on the three dimen-sions of social disorganization using the1994 BCS. For comparative purposes (whichwill be explicitly addressed below), the WLScoefficients from Sampson and Groves’s(1989) analysis of the 1982 BCS data arereprinted in Table 1 as well. Consistent withthe propositions set forth by Sampson andGroves, the first model indicates that SES,ethnic heterogeneity, and urbanization all exert a statistically significant inverseeffect on local friendship networks. Family dis-ruption was unrelated to local friendshipnetworks, yet residential stability was positively

408 CRIMINOLOGICAL THEORY: A TEXT/READER

Table 1Weighted Least Squares Regression Standardized Parameter Estimates of the Effects ofStructural Variables on Measures of Social Disorganization (n = 600)

Structural Variable

Local Friendship Networks

UnsupervisedPeer Groups

OrganizationalParticipation

1982 1994 1982 1994 1982 1994

Socioeconomic status −.06 −.11** −.34*** −.23*** .17** .07

Ethnic heterogeneity .02 −.27*** .13** .09** −.06 −.18***Residential stability .42*** .26*** .12* .04 −.09 .16***Family disruption −.03 −.02 .22*** .08* −.02 −.09**Urbanization −.27*** −.11** .15** .29*** −.10 .03R-Square .26*** .22*** .30*** .28*** .07*** .10***

*p < .10. **p < .05. ***p < .01.

Section VI � Replicating Sampson and Grove’s Test of Social Disorganization Theory 409

and significantly related to this indicator ofsocial disorganization.

The WLS model predicting unsupervisedpeer groups follows a similar pattern. Levels ofethnic heterogeneity, family disruption, andurbanization were all positively and signifi-cantly related to the existence of unsupervisedpeer groups. Unlike the previous model, how-ever, residential stability was not a statisti-cally significant predictor of this dimension of social disorganization in this model.Nevertheless, also consistent with social disor-ganization theory, the effect of SES on unsu-pervised peer groups was inverse and statisticallysignificant.

The WLS model predicting the social dis-organization measure of organizational par-ticipation is not quite as robust as theprevious two models. Compared to the R-square values of the models predictinglocal friendship networks and unsupervisedpeer groups (.22 and .28, respectively), the R-square for the model predicting organiza-tional participation (.10) is considerablysmaller. Even so, consistent with social disor-ganization theory, ethnic heterogeneity, resi-dential stability, and family disruption wereall significantly related to organizational par-ticipation in the theoretically expected direc-tion. Inconsistent with the propositions madeby Sampson and Groves (1989), however, SESand urbanization failed to significantly pre-dict levels of organizational participation.

Social Disorganizationand Total Victimization

Table 2 contains the results of the WLSmodel using the structural characteristics andthe social disorganization measures to pre-dict total criminal victimization. Two issues are important when examining this model: (1) whether the social disorganization measuressignificantly predict total criminal victimiza-tion, and (2) whether the social disorganization

measures mediate the effects of the structuralcharacteristics on total victimization.

With regard to the first issue, all three ofthe social disorganization measures were sig-nificantly related to total criminal victimiza-tion in the theoretically expected direction.The effect is particularly strong for the rela-tionship between unsupervised peer groupsand total victimization. These results are gen-erally supportive of the social disorganizationmodel articulated by Sampson and Groves(1989). The issue of the mediating effects ofthe social disorganization variables, on theother hand, offers less support for the socialdisorganization paradigm. In particular, aftercontrolling for the measures of social disorga-nization, the structural characteristics of resi-dential stability and family disruption stillmaintained statistically significant relation-ships with total victimization. Alternatively,on controlling for the social disorganizationmeasures, SES, ethnic heterogeneity, andurbanization failed to significantly predicttotal victimization.

Table 2

Weighted Least Squares RegressionStandardized Parameter Estimates ofthe Effects of Structural Variables andMeasures of Social Disorganizationon Total Victimization Rates (n = 600)

Independent Variable

Total Victimization Rate

1982 1994

Socioeconomic status –.03 .06Ethnic heterogeneity .08 .06Residential stability .03 –.06Family disruption .20*** .17***Urbanization .21*** .03Local friendship networks –.12** –.14***Unsupervised peer groups .34*** .40***Organizational participation –.11** –.06R-square .42*** .28***

**p < .05. ***p < .01.

Summary of 1994 BCS Replication

Our test of social disorganization theoryusing the 1994 BCS data is, on balance, sup-portive of the major propositions made bythe theory in three respects. First, each of thefive structural characteristics significantlypredicted at least two of the three social dis-organization measures specified by Sampsonand Groves (1989). Second, each of the socialdisorganization measures was significantlyrelated to rates of total criminal victimiza-tion. Finally, although the predicted relation-ships did not always occur, in the majority ofcases, the social disorganization measures didmediate the effects of the structural charac-teristics on levels of total criminal victimi -zation While this analysis does provideconsiderable support for the social disorgani-zation perspective, it still remains to be seenwhether the results found here are more orless consistent with those found in Sampsonand Groves’s analysis. In short, it is necessaryto empirically explore the degree to which theresults generated by the analysis of the 1982BCS data differ from our replication studyusing the 1994 BCS data.

Statistical Comparisons Between the 1982 and 1994 Parameter Estimates

Using the results from the 1994 BCSanalysis, 95 percent confidence intervals wereconstructed around each of the standardizedparameter estimates from the WLS regressionmodels. Once the upper and lower boundariesof the confidence intervals were established,we could then determine whether the parame-ter estimates from our replication were signifi-cantly different from Sampson and Groves’s(1989) analyses of the 1982 BCS data (i.e., thecoefficients presented in Table1 and Table 2).5

In essence, this series of significance tests con-tained two main objectives. First, they were

intended to reveal whether the results ofSampson and Groves’s (1989) original studywould, for the most part, be reproduced usinga different data set (i.e., to determine whetherthe results would be consistent across datasources). Second, if significant differences didemerge, these tests were also designed touncover whether such differences do, or donot, support social disorganization theory (i.e.,do the significant differences between coeffi-cients indicate more or less support for thetheory?).

Structural Characteristicsand Social Disorganization

Table 3 contains the results of the signifi-cance tests between the 1982 and 1994 BCScoefficients. Less than one-third of the coeffi-cient comparisons (7 of 23) indicated a statis-tically significant difference. For the most part,these differences were found in the effects ofthe structural characteristics on the social dis-organization variables. It is important to note,however, that none of these significant differ-ences are inconsistent with social disorganiza-tion theory.

The effects of residential stability andurbanization on local friendship networkswere smaller in the 1994 sample (yet still inthe expected directions), but the effect ofethnic heterogeneity is significantly strongerin the 1994 analysis. The structural predic-tors of unsupervised peer groups followed asimilar trend: the effect of family disruptionwas about one-third as strong in the 1994sample (yet still in the expected direction),but the influence of urbanization was nearlytwice as strong in the 1994 analysis. Only theeffect of residential stability was significantlydifferent for the structural predictors oforganizational participation. This relation-ship was nonsignificant in the 1982 analysis(and the direction of this coefficient was neg-ative), yet the 1994 data show that residential

410 CRIMINOLOGICAL THEORY: A TEXT/READER

Section VI � Replicating Sampson and Grove’s Test of Social Disorganization Theory 411

stability has a positive and statistically significant relationship with organizationalparticipation.

Social Disorganization and Total Victimization

As for the full model predicting totalcriminal victimization, only the effect ofurbanization was significantly different fromthe 1982 to the 1994 BCS data. In Sampsonand Groves’s (1989) analysis, the direct effectof urbanization on total victimization was notfully mediated by the social disorganizationvariables. The magnitude of the urbanizationcoefficient in the 1994 analysis, however, isabout one-seventh of the size of the parameterestimate from the 1982 sample. This findingfrom the 1994 BCS data indicates greater sup-port for social disorganization theory sincethe direct effect of this structural characteris-tic is even more dampened by the presence ofthe social disorganization variables.

As an added check on the direct andindirect effects of the structural characteris-tics on the total victimization rate, consis-tent with Veysey and Messner’s (1999)analysis of the 1982 BCS data we reestimatedthe full model in LISREL. As can be seen inTable 4, these results largely conform to thepropositions set forth by Sampson andGroves (1989). To be sure, the overall modelprovides a relatively good fit to the data,with a goodness-of-fit index (GFI) of .95and an adjusted goodness-of-fit index(AGFI) of .84—both of which are consistentwith Veysey and Messner’s (1999) resultswhen they allowed the error terms of the fivestructural characteristics to correlate (i.e.,their analytic improvement over the pathanalysis techniques employed by Sampsonand Groves).6 Even so, it is noteworthy thatmuch of the total effects of the structuralcharacteristics on the total victimization rateare indirect through the social disorganiza-tion variables.

Table 3Comparison of Standardized Weighted Least Squares Regression Coefficients Across 1982and 1994 British Crime Survey Samples

Friendship Unsupervised Organized Total Variable Networks Peer Groups Participation Victimization

Structural characteristics

Socioeconomic status NS NS NS NS

Ethnic heterogeneity Siga NS NS NS

Residential stability Siga NS Siga NS

Family disruption NS Siga NS NS

Urbanization Siga Siga NS Siga

Social disorganization measures

Local friendship networks — — — NS

Unsupervised peer groups — — — NS

Organizational participation — — — NS

a. Standardized parameter estimates differ significantly, yet the estimate generated by the 1994 British Crime Survey sample isconsistent with social disorganization theory.

Summary on 1982 and1994 BCS Comparisons

Our replication of the test of social disor-ganization theory conducted by Sampson andGroves (1989), using the 1994 BCS data, gen-erally mirrors the results that were found usingthe 1982 BCS data. Like Sampson and Groves’sstudy, our analysis reveals a relatively high levelof empirical support for the social disorganiza-tion perspective. Furthermore, the magnitudesof the parameter estimates across the two sam-ples are generally similar. Indeed, significantdifferences between the coefficients across thetwo data sets appeared in fewer than one-thirdof the comparisons. Notably, in each instancewhere the parameter estimates from the 1994sample did differ significantly from the 1982data source, the results generated from themore recent sample indicated a greater degreeof support for social disorganization theorythan was previously revealed. On balance,therefore, our analysis provides both empiricalsupport for the social disorganization perspec-tive and support for the conclusion thatSampson and Groves’s results were not idio-syncratic to the 1982 BCS data.

yy DiscussionCriminologists often embrace particular theo-retical explanations for reasons that have littleto do with a given theory’s empirical validity(Cole 1975; Gould 1996; Hagan 1973; Lilly,Cullen, and Ball 2002; Pfohl 1985; Pratt andCullen 2000). In this regard, social disorgani-zation theory’s “second wind” could be due, atleast in part, to the emergence of growing pub-lic concern over the patterns of “incivility” and“disorder” that characterize certain portions ofhigh-crime urban areas (Beckett 1997; DiIulio1997; Skogan 1992; Wilson and Kelling 1982).Nevertheless, the empirical validation thatSampson and Groves’s (1989) study accordedthe social disorganization perspective clearlywas integral to the perspective’s revitalization.

Although subsequent studies have alsorevealed a certain measure of support for themajor tenets of social disorganization theory(Bellair 1997; and Bergen and Herman 1998;Bursik and Grasmick 1993; Krivo and Peterson1996; Messner and Sampson 1991; Miethe,Hughes, and McDowall 1991; Morenoff andSampson 1997; Sampson and Raudenbush1999; Sampson et al. 1997; Warner and Pierce

412 CRIMINOLOGICAL THEORY: A TEXT/READER

Table 4Direct and Indirect Effects of Structural Characteristics and Social Disorganization Variableson the Total Victimization Rate Using LISREL

Variable Direct Effect Indirect Through Indirect Effect Total

Socioeconomic status .060 Organizational participation –.001 –.081

Unsupervised teen groups –1.40

Ethnic heterogeneity .060 Unsupervised teen groups .048 .108

Urbanization .030 Organizational participation .005 .057

Local friendship networks .022

Residential stability –.070 Local friendship networks –.052 –.122

Family disruption .170 Unsupervised teen groups .056 .226

Local friendship networks –.140 Unsupervised teen groups –.012 –.152

Organizational participation –.060 Unsupervised teen groups –.016 –.076

Unsupervised teen groups .400 — — .400

GFI = .95. AFGI = .84.

Section VI � Replicating Sampson and Grove’s Test of Social Disorganization Theory 413

1993; Warner and Roundtree 1997), none haveattempted to replicate Sampson and Groves’s(1989) analysis systematically and with a dif-ferent data set. This gap in the research litera-ture is potentially problematic. One does notneed not look terribly far to see the risks inher-ent in accepting a theoretical premise asvalid—and constructing a set of social policiesconsistent with the theory—based on theresults of a single study that may have pro-duced results that were unique to a particularpoint in time and space.7 More generally, thereproducibility of findings is a key feature ofscience—an activity that lacks the luster of dis-covering a new idea but is the linchpin toestablishing whether initial claims should be asaccepted as “knowledge” (Gendreau in press;Hall 2000). In this context, and given theextant void in the criminological literature,our analysis was intended to assess whether theresults of Sampson and Groves’s (1989) studyreflected time-specific phenomena or anenduring social reality. Upon comparing thefindings of our analyses of the 1994 BCS datato those of Sampson and Groves’s modelsassessing the 1982 wave of the BCS, four con-clusions can be drawn.

First, independent of any comparisons toSampson and Groves’s (1989) work, our analy-sis can be viewed on its own as having revealeda high level of support for social disorganiza-tion theory. Our measures of local friendshipnetworks, unsupervised peer groups, andorganizational participation effectively medi-ated—to a large extent—the relationshipsbetween certain structural characteristics ofneighborhoods and rates of criminal victim-ization. These measures of social disorganiza-tion were, in turn, significantly related to thetotal victimization rate (with the exception oforganizational participation). Thus, the majorpropositions specified by social disorganiza-tion theory—that certain structural character-istics of communities affect the ability ofresidents to impose social control mechanismsover their members, and that the loss of such

control mechanisms affects rates of crime—are supported here.

Second, the results of our replication aregenerally consistent with those found in theoriginal study by Sampson and Groves (1989).Indeed, the major findings related to thedirect, indirect, and mediating effects of com-munity-level structural characteristics andsocial disorganization variables are generallyreproduced here with a different data set. Ourreplication, therefore, indicates a pattern ofconsistency among the relationships specifiedby Sampson and Groves across the 1982 and1994 BCS data sets.

Third, not all of the relationships revealedin the original work of Sampson and Groves(1989) were identically replicated using the1994 BCS data. In particular, nearly a third ofthe parameter estimates for the relationshipsspecified and tested in the original study andour replication differed significantly. Even so, itis important to note that when parameter esti-mates did differ significantly, they all divergedin a direction that was consisted with social dis-organization theory (i.e., stronger mediatingeffects of the social disorganization variableson the relationships between the structuralcharacteristics and total victimization).

Fourth, when taken together, these factorsindicate that the theoretical attention that hasbeen accorded to Sampson and Groves’s(1989) “Community Structure and Crime:Testing Social-Disorganization Theory” is war-ranted. Although we cannot specify the degreeto which these findings would generalize tosocial settings outside of Britain, our resultssuggest that Sampson and Groves’s studyappears to have captured an ongoing empiricalreality. In short, our replication reinforces theview that Sampson and Groves’s essay deservesthe title of a “criminological classic.”

In concluding that the analysis providesadded support for social disorganization the-ory, however, we are not arguing for theoreticalstagnation. Thus, compared to the measuresused to assess the intervening or mediating

variables in our study, additional or morefinely calibrated measures of social disorgani-zation might account more extensively for thedirect effects of structural antecedents. But thealternative possibility is that the direct effectsreflect causal processes that other theories arebetter suited to explain. Furthermore, as othershave pointed out (Veysey and Messner 1999),the effects mediating disorganization factors inSampson and Groves’s model may themselvesbe open to alternative theoretical interpreta-tion. Sampson and Groves chose to interpretsocial disorganization theory as a perspectivethat linked crime rates to the breakdown ofinformal social control. Other scholars, how-ever, have argued that Shaw and McKay (1972)offered a “mixed model” of delinquency, whichtied crime both to the weakening of controland to the transmission and learning of crimi-nal values (Kornhauser 1978). Mediating vari-ables, such as unsupervised peers, thus mightcapture not only the attenuation of control butalso the presence of criminal subcultures andsocial learning (Veysey and Messner 1999).Further research is needed to reveal more pre-cisely why variables have the effects they doand the implications of these finding for com-peting theories of crime.

Equally salient, the social disorganizationperspective itself is evolving. Perhaps the mostnoteworthy advance is Sampson et al’s (1997)introduction of the concept of “collective effi-cacy.” At times, it appears that the concept ofcollective efficacy is the “opposite” of social dis-organization (Taylor 2001:128). However, thetheoretical advance comes in at least threeways. First, the concept of efficacy involves notonly informal control but also “trust” or “socialcohesion” (Sampson et al. 1997:920)—or whatmight be called “social support” (Cullen 1994).That is, control and trust are interrelated andmutually reinforcing. Second, the notion of“efficacy” implies a resource that can be acti-vated. The willingness to apply controls isfound in social disorganization theory as well,

but in Sampson et al.’s (1997) work, the choiceof the word efficacy implies more explicitly thatcollectives differ in their ability to actively fightagainst neighborhood conditions that arecriminogenic. Efficacy is not just a static stateof being but capital that can be “called intoplay” when needed (e.g., to clear drug pushersfrom a community playground).Third, the col-lective efficacy model moves beyond social dis-organization theory’s traditional neglect ofstructural inequality (Pfohl 1985), and seesantecedent variables not as separate entities butas interrelated. In particular, they link collectiveefficacy to the concentration of disadvantageand advantage (Sampson et al. 1997, 1999).

In many ways, these theoretical extensionsare to be welcomed. Nonetheless, three observa-tions are warranted. First, collective efficacyencapsulates the core elements of social disorga-nization, with the model including informalsocial control in its measures and conceptualiza-tion of efficacy. Accordingly, studies providingempirical confirmation for social disorganiza-tion theory—such as our analyses and Sampsonand Groves’s (1989) original work—lend indi-rect support for collective efficacy theory.Second, it has yet to be shown that collective effi-cacy theory is empirically superior to social disorganization theory. Unlike tests of individual-level theories (Hirschi 1969), macro-level theo-ries only rarely are tested against one another.Social disorganization theory—especially iftested in all its complexity as a ‘mixed model”—might rival collective efficacy in its ability toexplain variation in crime rates. Third, if socialdisorganization theory is the product of 1920sChicago (Lilly et al. 2002; Pfohl 1985), collectiveefficacy theory is the product of its time—cur-rent-day Chicago (Sampson et al. 1997). Itremains to be seen whether this perspective willhave more explanatory power than social disor-ganization theory when applied to other con-texts, including, for example, urban areas locatedelsewhere in the United States, in Europe, and inrapidly growing developing nations.

414 CRIMINOLOGICAL THEORY: A TEXT/READER

Section VI � Replicating Sampson and Grove’s Test of Social Disorganization Theory 415

yy Notes1. According to the Social Sciences Citation

Index, Sampson and Groves’s 1989 publication hasbeen cited 232 times, including 96 times since 1999and 32 times in the past 14 months. In a recent article,Wright, Malia, and Johnson (1999) reviewed the mostcited criminology and criminal justice works in 107articles from the leading sociology journals. Sampsonand Groves’s publication ranked third, higher than anumber of noteworthy works (e.g., Black 1976; Blauand Blau 1982; Cohen and Felson 1979; Gottfredsonand Hirschi 1990; Kornhauser 1978; Merton 1968).

2. Sampson and Groves (1989) focused primarilyon the 1982 BCS data. They supplemented their analy-sis with the 1984 BCS data, and the results were largelyconsistent with those from the 1982 sample. Thus,consistent with Sampson and Groves’ original focus, inthe present study, we compare our results to theiranalyses of the 1982 BCS data.

3. We should note that in their original article,Sampson and Groves (1989) attempted to confirmtheir 1982 analysis with a partial replication of datadrawn from the 1984 BCS (e.g., there was no measureof organizational participation). Their results wereconsistent with the findings drawn from the 1982analysis and lent added support to social disorganiza-tion theory. Our replication is patterned after the morecomplete 1982 study. In either case, however, our inde-pendent replication is conducted on BCS data col-lected a full decade after the surveys analyzed bySampson and Groves.

5. Traditional methods for comparing the equal-ity of coefficients (see, e.g., Brame et al. 1998; Clogg,Petkova, and Haritou 1995; Cohen 1983) require theuse of both the metric parameter estimates and thestandard errors from each model. Since we only havesuch information (specifically, the standard errors) forour 1994 analysis, the use of confidence intervals forthis purpose is the next methodologically rigorousmethod at our disposal.

6. Veysey and Messner (1999) did, however, arriveat a model with a Goodness-of-Fit Index of .988 and anAdjusted Goodness-of-Fit Index of .939 when they freedthe parameters to estimate direct effects for socioeco-nomic status, ethnic heterogeneity, residential stability,and family disruption on the total victimization rate.

7. For example, in their classic Minneapolisexperiment, Sherman and Berk (1984) found that

arrest had a deterrent effect on domestic violencerecidivism. Subsequent replications, however, pro-duced inconsistent results and the specification thatarrest primarily had effects on offenders with socialbonds (see, e.g., Dunford, Huizinga, and Elliott 1990;Sherman and Berk 1984; Sherman et al. 1991, 1992).

yy ReferencesAkers, Ronald L. 1999. Criminological Theories:

Introduction and Evaluation. 2nd ed. Chicago:Fitzroy Dearborn.

Beckett, Katherine. 1997. Making Crime Pay: Law andOrder in Contemporary American Politics. NewYork: Oxford University Press.

Bellair, Paul E. 1997. “Social interaction andCommunity Crime: Examining the Importance of Neighborhood Networks.” Criminology 35: 677–703.

Bergen, Albert, and Max Herman. 1998. “Immigration,Race, and Riot: The 1992 Los Angeles Uprising.”American Sociological Review 63: 39–54.

Black, Donald. 1976. The Behavior of Law. New York:Academic Press.

Blau, Judith R., and Peter M. Blau. 1982. “The Cost ofInequality: Metropolitan Structure and ViolentCrime.” American Sociological Review 47: 114–29.

Brame, Robert, Raymond Paternoster, Paul Mazerolle,and Alex Piquero. 1998. “Testing for the Equalityof Maximum-Likelihood Regression CoefficientsBetween Two Independent Equations.” Journal ofQuantitative Criminology 14: 245–61.

Bursik, Robert J. 1986. “Delinquency Rates as Sourcesof Ecological Change.” Pp. 63–74 in The SocialEcology of Crime, edited by James M. Byrne andRobert J. Sampson. New York: Springer-Verlag.

_____. 1988. “Social Disorganization and Theories ofCrime and Delinquency: Problems and Prospects.”Criminology 26: 519–51.

Bursik, Robert J., and Harold G. Grasmick. 1993.“Economic Deprivation and Neighborhood CrimeRates, 1960–1980.” Law and Society Review 27:263–83.

Byrne, James M., and Robert J. Sampson. 1986. “KeyIssues in the Social Ecology of Crime.” Pp. 1–22 inThe Social Ecology of Crime, edited by James M.Byrne and Robert J. Sampson. New York: Springer-Verlag.

Clogg, Clifford C., Eva Petkova, and AdamantiosHaritou. 1995. “Statistical Methods for ComparingRegression Coefficients Between Models.”American Journal of Sociology 100: 1261–93.

Cohen, Ayala. 1983. “Comparing Regression CoefficientsAcross Subsamples: A Study of the Statistical Test.”Sociological Methods and Research 12: 77–94.

Cohen, Lawrence E., and Marcus Felson. 1979. “SocialChange and Crime Rate Trends: A Routine ActivityApproach.” American Sociological Review 44: 588–608.

Cole, Stephen. 1975. “The Growth of ScientificKnowledge: Theories of Deviance as a CaseStudy.” Pp. 175–200 in The Idea of Social Structure:Papers in Honor of Robert K. Merlon, edited byLewis A. Coser. New York: Harcourt BraceJovanovich.

Cullen, Francis T. 1994. “Social Support as anOrganizing Concept for Criminology: PresidentialAddress to the Academy of Criminal JusticeSciences.” Justice Quarterly 11: 527–59.

DiIulio, John J. 1997. “Are Voters Fools? Crime, PublicOpinion, and Democracy.” Corrections Management Quarterly 1 (3): 1–5.

Dunford, Franklyn, David Huizinga, and Delbert S.Elliott 1990. “The Role of Arrest in DomesticAssault: The Omaha Experiment.” Criminology 28:183–206.

Gendreau, Paul. In press. “We Must Do a Better Job ofCumulating Knowledge.” Canadian Psychology.

Gottfredson, Michael R., and Travis Hirschi. 1990. AGeneral Theory of Crime. Stanford, CA: StanfordUniversity Press.

Gould, Stephen Jay. 1996. The Mismeasure of Man. Rev.ed. New York: W. W. Norton.

Hagan, John. 1973. “Labeling and Deviance: A CaseStudy in the ‘Sociology of the Interesting.’” SocialProblems 20: 447–58.

Hall Nancy S. 2000. “The Key Role of Replication inScience.” The Chronicle of Higher Education,November 10, B14.

Hirschi, Travis, 1969. Causes of Delinquency. Berkeley:University of California Press.

Home Office Crime and Criminal Justice Unit Officeof Population Census and Social Surveys Division.1994. British Crime Survey (computer file).Colchester, Essex, UK: The Data Archive.

Kasarda, John D., and Morris Janowitz. 1974.“Community Attachment in Mass Society.”American Sociological Review 39: 328–39.

Kempf, Kimberly L. 1993. “The Empirical Status ofHirschi’s Control Theory.” Pp. 143–85 in NewDirections in Criminological Theory: Advances inCriminological Theory, Vol. 4, edited by Freda Adlerand William S. Laufer. New Brunswick, NJ:Transaction.

Kornhauser, Ruth R. 1978. Social Sources ofDelinquency: An Appraisal of Analytic Models.Chicago: University of Chicago Press.

Krivo, Lauren J., and Ruth D. Peterson. 1996.“Extremely Disadvantaged Neighborhoods andUrban Crime.” Social Forces 75: 619–50.

Krohn, Marvin D. 1986. “The Web of Conformity: ANetwork Approach to the Explanation ofDelinquent Behavior.” Social Problems 33: 81–93.

Lilly, Robert J., Francis T. Cullen, and Richard A. Ball.2002. Criminological Theory: Context andConsequences. 3rd ed. Thousand Oaks, CA: Sage.

Lynn, Peter. 1997. “Sampling Frame Effects on theBritish Crime Survey.” Journal of the RoyalStatistical Society Series 160 (2): 253–69.

Merton, Robert K. 1968. Social Theory and SocialStructure. New York: Free Press.

Messner, Steven F., and Robert J. Sampson. 1991. “TheSex Ratio, Family Disruption, and Rates of ViolentCrime: The Paradox of Demographic Structure.”Social Forces 69: 693–713.

Miethe, Terance D., Michael Hughes, and DavidMcDowall. 1991. “Social Change and Crime Rates:An Evaluation of Alternative TheoreticalApproaches.” Social Forces 70: 165–85.

Morenoff, Jeffrey D., and Robert J. Sampson. 1997.“Violent Crime and the Spatial Dynamics ofNeighborhood Transition: Chicago, 1970-1990.”Social Forces 76: 31–64.

Morenoff, Jeffrey D., Robert J. Sampson, and StephenW. Raudenbush, 2001. “Neighborhood Inequality,Collective Efficacy, and the Spatial Dynamics ofUrban Violence.” Criminology 39: 517–59.

Pfohl, Stephen J. 1985. Images of Deviance and SocialControl: A Sociological History. New York:McGraw-Hill.

Pratt, Travis C., and Francis T. Cullen. 2000. “TheEmpirical Status of Gottfredson and Hirschi’sGeneral Theory of Crime: A Meta-Analysis.”Criminology 38: 931–64.

Roundtree, Pamela Wilcox, and Kenneth C. Land.1996. “Burglary Victimization, Perceptions ofCrime Risk, and Routine Activities: A MultilevelAnalysis Across Seattle Neighborhoods and

416 CRIMINOLOGICAL THEORY: A TEXT/READER

Section VI � Replicating Sampson and Grove’s Test of Social Disorganization Theory 417

Census Tracts.” Journal of Research in Crime andDelinquency 33: 147–80.

Roundtree, Pamela Wilcox, and Barbara D. Warner.1999. “Social Ties and Crime: Is the RelationshipGendered?” Criminology 37: 789–812.

Sampson, Robert J. 1987. “Urban Black Violence: TheEffect of Male Joblessness and Family Disruption.”American Journal of Sociology 93: 348–82.

Sampson, Robert J., and W. Byron Groves. 1989.“Community Structure and Crime: Testing Social-Disorganization Theory.” American Journal ofSociology 94: 774–802.

Sampson, Robert J., Jeffery D. Morenoff, and FeltonEarls. 1999. “Beyond Social Capital: SpatialDynamics of Collective Efficacy for Children.”American Sociological Review 64: 633–60.

Sampson, Robert J., and Stephen W. Raudenbush.1999. “Systematic Social Observation of PublicSpaces: A New Look at Disorder in UrbanNeighborhoods.” American Journal of Sociology105: 603–51.

Sampson, Robert J., Stephen W. Raudenbush, andFelton Earls. 1997. “Neighborhoods and ViolentCrime: A Multilevel Study of Collective Efficacy.”Science 277: 918–24.

Shaw, Clifford R., and Henry D. McKay. 1972. JuvenileDelinquency and Urban Areas. Chicago: Universityof Chicago Press.

Sherman, Lawrence W., and Richard A. Berk. 1984.“The Specific Deterrent Effects of Arrest forDomestic Assault.” American Sociological Review49: 261–72.

Sherman, Lawrence W., Janell D. Schmidt, Dennis P.Rogan, Patrick R. Gartin, Ellen G. Cohn, Dean J.Collins, and Anthony R. Bacich. 1991. “FromInitial Deterrence to Long-Term Escalation: Short-Custody Arrest for Poverty Ghetto DomesticViolence.” Criminology 29: 821–50.

Sherman, Lawrence W., Douglas A. Smith, Janell D.Schmidt, and Dennis P. Rogan. 1992. “Crime,Punishment, and Stake in Conformity; Legal andInformal Control of Domestic Violence.” AmericanSociological Review 57: 680–90.

Siegel, Larry. 2000. Criminology. 7th ed. Belmont, CA:Wadsworth.

Skogan, Wesley G. 1992. Disorder and Decline: Crime andthe Spiral of Decay in American Neighborhoods.Berkeley: University of California Press.

Sykes, Gresham M., and Francis T. Cullen. 1992.Criminology. 2nd ed. New York: Harcourt BraceJovanovich.

Taylor, Ralph B. 2001. “The Ecology of Crime, Fear,and Delinquency: Social Disorganization VersusSocial Efficacy. In Explaining Criminals and Crime:Essays in Contemporary Criminological Theory,edited by Raymond Paternoster and RonetBachman. Los Angeles: Roxbury.

Tittle, Charles R., and Raymond Paternoster. 2000.Social Deviance and Crime: An Organizational andTheoretical Approach. Los Angeles: Roxbury.

Velez, Maria B. 2001. “The Role of Public SocialControl in Urban Neighborhoods: A Multi-LevelAnalysis of Victimization Risk.” Criminology 39:837–64.

Veysey, Bonita M., and Steven F. Messner. 1999.“Further Testing of Social Disorganization Theory:An Elaboration of Sampson and Groves’s‘Community Structure and Crime’.” Journal ofResearch in Crime and Delinquency 36: 156–74.

Warner, Barbara D., and Glenn L. Pierce. 1993.“Reexamining Social Disorganization TheoryUsing Calls to the Police as a Measure of Crime.”Criminology 31: 493–517.

Warner, Barbara D., and Pamela Wilcox Roundtree.1997. “Local Ties in a Community and CrimeModel: Questioning the Systemic Nature ofInformal Social Control.” Social Problems 44: 520–36.

Wilson, James Q., and George L. Kelling. 1982.“Broken Windows: The Police and NeighborhoodSafety.” Atlantic Monthly, March, 29–38.

Wright, Richard A., Michael Malia, and C. WayneJohnson. 1999. “Invisible Influence: A CitationAnalysis of Crime and Justice Articles Published inLeading Sociology Journal.” Journal of Crime andJustice 22: 147–65.

R E V I E W Q U E S T I O N S

1. Identify and discuss the three types of variables used by Lowenkamp et al. (as well as Sampson andGrove) to measure the key intervening variables in the model of social disorganization. Which of thesethree do you feel is the best measure of community (or informal) controls/organization, and why?Which of these do you feel is the least applicable, and why?

418 CRIMINOLOGICAL THEORY: A TEXT/READER

2. Identify and discuss the five types of structural variables of neighborhoods that Lowenkamp et al. (aswell as Sampson and Grove) claim are mediated by the intervening variables (which you discussed inReview Question #1). Which of these structural variables do you think is most vital in predicting crimerates of neighborhoods, and why? Which structural variable do you think is least vital, and why?

3. If you were asked to make a conclusion as a social scientist regarding the current state of empirical evi-dence or support for Shaw and McKay’s proposed model of social disorganization, what would you con-clude? What key weaknesses do you see with the model or the way it has been tested?

4. To what extent do the findings presented by Lowenkamp et al. (or Samson and Grove) fit what you haveobserved in your own local communities?

R E A D I N G

In this selection, Marvin Wolfgang and Franco Ferracuti propose a theoretical model thatexplains how an entire other culture (e.g., subculture) can exist, even within an existing norma-tive framework by a majority of individuals in a society. The authors propose a “subculture ofviolence,” which they note is predominately held by young males. They argue that violence is atheme in the “values that make up the life-style, the socialization process, the interpersonal rela-tionships of individuals living in similar conditions.” This selection reveals the core elements ofthis theory, including the social learning and cultural context of this subculture, as well as con-clusions that can be made regarding the existence of such subcultures. While reading this selec-tion, readers are encouraged to think about people they know or grew up with and consider ifthey can relate adherence to subcultural norms regarding violence to anyone in their past.

The Subculture of Violence

Marvin E. Wolfgang and Franco Ferracuti

yy The Thesis of aSubculture of Violence

This section examines the proposition thatthere is a subculture of violence . . . to extend atheoretical formulation regarding the existence

of subcultures in general if we are to hypothe-size a particular subculture of violence. Itwould be difficult to support an argument thata subculture exists in relation to a single cul-tural interest, and the thesis of a subculture ofviolence does not suggest a monolithic character.

SOURCE: Marvin E. Wolfgang and Franco Ferracuti, The Subculture of Violence (London: Routledge Publishers, 1967), 140-61.Copyright 1967 Routledge. Reproduced by permission of Taylor & Francis Books UK.

Section VI � The Subculture of Violence 419

It should be remembered that the term itself—subculture—presupposes an already existingcomplex of norms, values, attitudes, materialtraits, etc. What the subculture-of-violenceformulation further suggests is simply thatthere is a potent theme of violence current inthe cluster of values that make up the life-style,the socialization process, the interpersonalrelationships of individuals living in similarconditions.

The analysis of violent aggressive behaviorhas been the focus of interest of many socialand biological researchers, and psychology hasattempted to build several theories to explainits phenomenology, ranging from the death-aggression instinct of the psychoanalyticschool to the frustration-aggression hypothe-sis. The present discussion is the result of jointexplorations in theory and research in psy-chology and in sociology, using the concept ofsubculture as a learning environment. Ourmajor area of study has been assaultive behav-ior, with special attention to criminal homi-cide . . . [S]ome of the main trends incriminological thinking related to this topicmust now be anticipated for the proper focusof the present discussion.

Isolated sectional studies of homicidebehavior are extremely numerous, and it is notour intention to examine them in this chapter.There are basically two kinds of criminal homi-cide: (1) premeditated, felonious, intentionalmurder; (2) slaying in the heat of passion, orkilling as a result of intent to do harm but with-out intent to kill. A slaying committed by onerecognized as psychotic or legally insane, or bya psychiatrically designated abnormal subjectinvolves clinical deviates who are generally notheld responsible for their behavior, and who,therefore, are not considered culpable. We areeliminating these cases from our present dis-cussion, although subcultural elements are notirrelevant to the analysis of their psychopatho-logical phenomenology.

Probably fewer than five per cent of allknown homicides are premeditated, planned

intentional killings, and the individuals whocommit them are most likely to be episodicoffenders who have never had prior contactwith the criminal law. Because they are rarecrimes often planned by rationally functioningindividuals, perhaps they are more likely toremain undetected. We believe that a type ofanalysis different from that presented heremight be applicable to these cases. Our majorconcern is with the bulk of homicides—thepassion crimes, the violent slayings—that arenot premeditated and are not psychotic mani-festations. Like Cohen, who was concernedprincipally with most delinquency that arisesfrom the “working-class” ethic, so we are focus-ing [on] the preponderant kind of homicide,although our analysis . . . will include much ofthe available data on homicide [in] general.

Social Learning and Conditioning

Studies of child-rearing practices in rela-tion to the development of aggressive traitslead us to consider briefly one important theo-retical development from the field of generaland social psychology which, in our opinion,provides the theoretical bridge between anindividual’s violent behavior and his subcul-tural value allegiance. We are referring to whatis generally included under the heading of‘social learning.’ Issues which arise in anyanalysis of the structure and phenomenologyof subcultures are the process of transmittingthe subculture values, the extent of individualdifferences in the strength of allegiance tothose values, and the fact that not all the indi-viduals with ecological propinquity sharevalue and motive identity with the surround-ing culture. The process of social learning,through a number of mechanisms rangingfrom repetitive contacts to the subtler forms ofimitation and identification, involves theacquisition of value systems in early childhoodand their integration in the complex personal-ity trait-value-motive system, which makes upthe adult global individuality.

A recent paper by Jeffery (1965) summa-rizes a number of theoretical statementsaccepting the general principle that criminalbehavior can be explained as learned behaviorif conceptualized as operant behavior andreinforced through reward and immediategratification. However, the complexity oflearning theory and the serious uncertaintiesthat still plague its core concepts have thus farproduced an heuristic deficiency in transfer-ring from theory and experimental laboratoryresearch to field applications. The same may besaid about transferring to diagnostically ori-ented studies of the differential psychology ofviolent offenders. Admittedly, the transposi-tion from laboratory and animal experimenta-tion to the street corner and the prison is noteasy, is somewhat speculative, and may proveimpossible until the social-learning approachcan produce measurable, economical, andvalid diagnostic instruments.

An interesting beginning towards such adevelopment in criminology has been made byEysenck and his collaborators. A generalrestatement of the theory can be found inTrasler, and earlier statements appear inMowrer, for example. Bandura’s rich produc-tion follows a social-learning approach, andhis recent books with Walters provide adetailed discussion of mechanisms, patterns,and implications for the application of abehavioristic learning approach to analysis ofpersonality development.

Indoctrination into a subculture can takeplace through early-infancy learning processes.However, not only does this indoctrinationprove difficult to reconstruct in an individualdiagnostic process, or impossible to demon-strate in the laboratory, but it is confused withindividual differences. These differentials in theimitation and identification processes beg thecentral question of why equally exposed indi-viduals terminally behave differently andexhibit values and norms that resist attempts toclassify them into discrete, yet uniform cate-gories. Eysenck has approached this problem

through introduction of the concept of individ-ual differences in conditioning, including, byextension, social conditioning. This approachassumes that, whereas introverts are easier tocondition and therefore more readily absorbsocialized values, extroverts are resistant to con-ditioning and dominated by anti-social impul-sive reactions. The conceptualization can beextended to include social learning of wholeantisocial value systems. These notions, if logi-cally followed, would postulate two types ofviolent offender: (1) the introversive, who aresocialized into a subculture of violence throughconditioning, and are frequent in specific eco-logical settings; (2) the extroversive, impulsive,unsocialized types, who cut across social, cul-tural, and subcultural strata. Both types canexhibit violent behavior, but the etiology andthe probability of such behavior vary along withbasic psychological make-up, i.e., a set of inher-ited characteristics which, in Eysenck’s terms,dichotomize individuals according to biologicaldeterminants that place them in a given posi-tion on the introversion-extroversion contin-uum. Only modest confirmation is so faravailable for this far-reaching conceptualiza-tion. An extension of the behavioristic learningtheory into therapy has been proposed byBandura and Walters and analyzed by severalothers. The advisability of granting scientificstatus to an approach which is still highly exper-imental has been seriously questioned.

Although the social-learning approachstill awaits confirmation, it does provide a con-ceptually useful bridge between the sociologi-cal, the psychological, and clinical constructswhich we have discussed in the precedingpages. It also furnishes us with the possibilityof utilizing two other personality theorieswhich have a definite place in the transpositionof the concept of subculture from sociology topsychology. Dissonance theory, as one of these,constitutes an elegant, if unproved, linkbetween the cognitive aspects of subculturalallegiance, the psychoanalytic mechanism ofprojection, and the internal consistency (with

420 CRIMINOLOGICAL THEORY: A TEXT/READER

Section VI � The Subculture of Violence 421

consequent reduction of the anxiety level)which constitutes the differential characteris-tics of members of the subculture. No disso-nance is experienced so long as the valuesystem of the individual is not confronted bydifferent or certainly conflicting values. Thetreatment implications of the concept of cogni-tive dissonance in relation to subculture alle-giance are obvious, and point to the need tofragment and rearrange antisocial groupalliances. The utilization of cognitive disso-nance in this way in the prevention of interna-tional conflict has been advocated, for example,by Stagner and Osgood. Stagner has, however,carefully analyzed the importance of perceptualpersonality theory to individual and groupaggression. A subculture allegiance entails anorganization or reorganization of the processof personality formation as a process of learn-ing to perceive objects, persons, and situationsas attractive or threatening, in accordance withsubcultural positive and negative valences.

The general psychological contributionsfrom conditioning, learning theory, cognitivedissonance, perceptual personality theory, areindeed far from providing a total theoretical sys-tem as a counterpart to the sociological notionsabout subcultures. However, we are convincedthat these behavioral constructs of social learn-ing not only are the most directly related to sub-culture theory but also are capable of generatingan integrated theory in criminology.

The Cultural Context

Like all human behavior, homicide andother violent assaultive crimes must be viewedin terms of the cultural context from which theyspring. De Champneuf, Guerry, Quetelet early inthe nineteenth century, and Durkheim later, ledthe way toward emphasizing the necessity toexamine the physique sociale, or social phenom-ena characterized by “externality,” if the scientistis to understand or interpret crime, suicide,prostitution, and other deviant behavior.Without promulgating a sociological fatalism,

analysis of broad macroscopic correlates in thisway may obscure the dynamic elements of thephenomenon and result in the empirical hiatusand fallacious association to which Selvin refers.Yet, because of wide individual variations, theclinical, idiosyncratic approach does not neces-sarily aid in arriving at Weber’s Verstehen, ormeaningful adequate understanding of regular-ities, uniformities, or patterns of interaction.And it is this kind of understanding we seekwhen we examine either deviation from, or con-formity to, a normative social system.

Sociological contributions have madealmost commonplace, since Durkheim, the factthat deviant conduct is not evenly distributedthroughout the social structure. There is muchempirical evidence that class position, ethnic-ity, occupational status, and other social vari-ables are effective indicators for predicting ratesof different kinds of deviance. Studies in ecol-ogy perform a valuable service for examiningthe phenomenology and distribution of aggres-sion, but only inferentially point to the impor-tance of the system of norms. Anomie, whetherdefined as the absence of norms (which is adoubtful conceptualization) or the conflict ofnorms (either normative goals or means), orwhether redefined by Powell as ‘meaningless-ness,’ does not coincide with most empiricalevidence on homicide. Acceptance of the con-cept of anomie would imply that marginalindividuals who harbor psychic anomie thatreflects (or causes) social anomie have thehighest rates of homicides. Available data seemto reject this contention. Anomie as cultureconflict, or conflict of norms, suggests, as wehave in the last section, that there is one seg-ment (the prevailing middle-class value sys-tem) of a given culture whose value system isthe antithesis of, or in conflict with, another,smaller segment of the same culture. This con-ceptualism of anomie is a useful tool for refer-ring to subcultures as ideal types, or mentalconstructs. But to transfer this norm-conflictapproach from the social to the individuallevel, theoretically making the individual a

repository of culture conflict, again does notconform to the patterns of known psychologi-cal and sociological data. This latter approachwould be forced to hypothesize that sociallymobile individuals and families would be mostfrequently involved in homicide, or that per-sons moving from a formerly embraced sub-value system to the predominant communalvalue system would commit this form of vio-lent deviation in the greatest numbers. Thereare no homicide data that show high rates ofhomicides among persons manifesting highersocial aspirations in terms of mobility. Itshould also be mentioned that anomie, as aconcept, does not easily lend itself to psycho-logical study.

That there is a conflict of value systems, weagree. That is, there is a conflict between a pre-vailing culture value and some subculturalentity. But commission of homicide by actorsfrom the subculture at variance with the pre-vailing culture cannot be adequately explainedin terms of frustration due to failure to attainnormative-goals of the latter, in terms of inabil-ity to succeed with normative-procedures(means) for attaining those goals, nor in termsof an individual psychological condition ofanomie. Homicide is most prevalent, or thehighest rates of homicide occur, among a rela-tively homogeneous subcultural group in anylarge urban community. Similar prevalent ratescan be found in some rural areas. The valuesystem of this group, we are contending, con-stitutes a subculture of violence. From a psy-chological viewpoint, we might hypothesizethat the greater the degree of integration of theindividual into this subculture, the higher theprobability that his behavior will be violent in avariety of situations. From the sociological side,there should be a direct relationship betweenrates of homicide and the extent to which thesubculture of violence represents a cluster ofvalues around the theme of violence.

Except for war, probably the most highlyreportable, socially visible, and serious form ofviolence is expressed in criminal homicide.

Data show that in the United States rates arehighest among males, non-whites, and theyoung adult ages. Rates for most seriouscrimes, particularly against the person, arehighest in these same groups. In a Philadelphiastudy of 588 criminal homicides, for example,non-white males aged 20–24 had a rate of 92per 100,000 compared with 3:4 [3.4] for whitesmales of the same ages. Females consistentlyhad lower rates than males in their respectiverace groups (non-white females, 9:3 [9.3];white females, 0:4 [0.4], in the same study),although it should be . . . that non-whitefemales have higher rates than white males.

It is possible to multiply these specific find-ings in any variety of ways; and although a sub-cultural affinity to violence appears to beprincipally present in large urban communitiesand increasingly in the adolescent population,some typical evidence of this phenomenon canbe found, for example, in rural areas andamong other adult groups. For example, a par-ticular, very structured, subculture of this kindcan be found in Sardinia, in the central moun-tain area of the island. Pigliaru has conducted abrilliant analysis of the people from this areaand of their criminal behavior, commonlyknown as the vendetta barbaricina. . . .

. . . . We suggest that, by identifying thegroups with the highest rates of homicide, weshould find in the most intense degree a sub-culture of violence; and, having focused onthese groups, we should subsequently examinethe value system of their subculture, theimportance of human life in the scale of val-ues, the kinds of expected reaction to certaintypes of stimulus, perceptual differences in theevaluation of stimuli, and the general person-ality structure of the subcultural actors. In thePhiladelphia study it was pointed out that:

the significance of a jostle, a slightlyderogatory remark, or the appearanceof a weapon in the hands of an adver-sary are stimuli differentially perceivedand interpreted by Negroes and whites,

422 CRIMINOLOGICAL THEORY: A TEXT/READER

Section VI � The Subculture of Violence 423

males and females. Social expectationsof response in particular types of socialinteraction result in differential “defini-tions of the situation.” A male is usuallyexpected to defend the name andhonor of his mother, the virtue ofwomanhood . . . and to accept no dero-gation about his race (even from amember of his own race), his age, or hismasculinity. Quick resort to physicalcombat as a measure of daring,courage, or defense of status appears tobe a cultural expression, especially forlower socio-economic class males ofboth races. When such a culture normresponse is elicited from an individualengaged in social interplay with otherswho harbor the same response mecha-nism, physical assaults, altercations,and violent domestic quarrels thatresult in homicide are likely to be com-mon. The upper-middle and uppersocial class value system defines subcul-tural mores, and considers many of thesocial and personal stimuli that evoke acombative reaction in the lower classesas “trivial.” Thus, there exists a culturalantipathy between many folk rational-izations of the lower class, and of malesof both races, on the one hand, and themiddle-class legal norms under whichthey live, on the other.

This kind of analysis, combined with otherdata about delinquency, lower-class socialstructure, its value system, and its emphasis onaggression, suggest the thesis of a violent sub-culture, or, by pushing the normative aspects alittle further, a subculture of violence. Amongmany juvenile gangs, as has repeatedly beenpointed out, there are violent feuds, meetings,territorial fights, and the use of violence toprove “heart,” to maintain or to acquire “rep.”

Physical aggression is often seen as ademonstration of masculinity and toughness.We might argue that this emphasis on showing

masculinity through aggression is not alwayssupported by data. If homicide is any index atall of physical aggression, we must rememberthat in the Philadelphia data non-white femaleshave [homicide] rates often two to four timeshigher than the rates of white males. Violentbehavior appears more dependent on culturaldifferences than on sex differences, tradition-ally considered of paramount importance inthe expression of aggression. . . .

. . . . It appears valid to suggest that thereare, in a heterogeneous population, differencesin ideas and attitudes toward the use of vio-lence and that these differences can be observedthrough variables related to social class andpossibly through psychological correlates.There is evidence that modes of control ofexpressions of aggression in children varyamong the social classes. Lower-class boys, forexample, appear more likely to be orientedtoward direct expression of aggression than aremiddle-class boys. The type of punishmentmeted out by parents to misbehaving childrenis related to this class orientation towardaggression. Lower-class mothers report thatthey or their husbands are likely to strike theirchildren or threaten to strike them, whereasmiddle-class mothers report that their type ofpunishment is psychological rather than physi-cal; and boys who are physically [punished]express aggression more directly than thosewho are punished psychologically. As Martinhas suggested, the middle-class child is morelikely to turn his aggression inward; in theextreme and as an adult he will commit suicide.But the lower-class child is more accustomed toa parent-child relationship which during pun-ishment is for the moment that of attacker andattacked. The target for aggression, then, isexternal; aggression is directed toward others.

The existence of a subculture of violence ispartly demonstrated by examination of the socialgroups and individuals who experience the high-est rates of manifest violence. This examinationneed not be confined to the study of one nationalor ethnic group. On the contrary, the existence of

a subculture of violence could perhaps receiveeven cross-cultural confirmation. Criminalhomicide is the most acute and highly reportableexample of this type of violence, but some circu-larity of thought is obvious in the effort to spec-ify the dependent variable (homicide), and alsoto infer the independent variable (the existenceof a subculture of violence). The highest rates ofrape, aggravated assaults, persistency in arrestsfor assaults (recidivism) among these groupswith high rates of homicide are, however, empir-ical addenda to the postulation of a subculture ofviolence. Residential propinquity of these samegroups reinforces the socio-psychological impactwhich the integration of this subculture engen-ders. Sutherland’s thesis of “differential associa-tion,” or a psychological reformulation of thesame theory in terms of learning process, couldeffectively be employed to describe more fullythis impact in its intensity, duration, repetition,and frequency. The more thoroughly integratedthe individual is into this subculture, the moreintensely he embraces its prescriptions of behav-ior, its conduct norms, and integrates them intohis personality structure. The degree of integra-tion may be measured partly and crudely by pub-lic records of contact with law, so high arrestrates, particularly high rates of assault crimes andrates of recidivism for assault crimes amonggroups that form the culture of violence, mayindicate allegiance to the values of violence.

We have said that overt physical violenceoften becomes a common subculturally expectedresponse to certain stimuli. However, it is notmerely rigid conformity to the demands andexpectations of other persons, as Henry andShort seem to suggest, that results in the highprobability of homicide. Excessive, compulsive,or apathetic conformity [of] middle-class indi-viduals to the value system of their social groupis a widely recognized cultural malady. Ourconcern is with the value elements of violenceas an integral component of the subculturewhich experiences high rates of homicide. It isconformity to this set of values, and not rigidconformity per se, that gives important meaningto the subculture of violence. . . .

It is not far-fetched to suggest that a wholeculture may accept a value set dependent uponviolence, demand or encourage adherence toviolence, and penalize deviation. . . . Homicide,it appears, is often a situation not unlike that ofconfrontations in wartime combat, in whichtwo individuals committed to the value of vio-lence come together, and in which chance,prowess, or possession of a particular weapondictates the identity of the slayer and of theslain. The peaceful non-combatant in both setsof circumstances is penalized, because of theallelomimetic behavior of the group support-ing violence, by his being ostracized as an out-group member, and he is thereby segregated(imprisoned, in wartime, as a conscientiousobjector) from his original group. If he is notsegregated, but continues to interact with hisoriginal group in the public street or on thefront line that represents the culture of vio-lence, he may fall victim to the shot or stabfrom one of the group who still embraces thevalue of violence. . . .

. . . . We have said that overt use of force orviolence, either in interpersonal relationshipsor in group interaction, is generally viewed as areflection of basic values that stand apart fromthe dominant, the central, or the parent culture.Our hypothesis is that this overt . . . (and oftenillicit) expression of violence (of which homi-cide is only the most extreme) is part of a sub-cultural normative system, and that this systemis reflected in the psychological traits of thesubculture participants. In the light of our dis-cussion of the caution to be exercised in inter-pretative analysis, in order to tighten the logicof this analysis, and to support the thesis of asubculture of violence, we offer the followingcorollary propositions:

1. No subculture can be totally differentfrom or totally in conflict with the societyof which it is a part.A subculture of vio-lence is not entirely an expression ofviolence, for there must be interlockingvalue elements shared with the domi-nant culture. . . .

424 CRIMINOLOGICAL THEORY: A TEXT/READER

Section VI � The Subculture of Violence 425

2. To establish the existence of a subcultureof violence does not require that theactors sharing in these basic value ele-ments should express violence in all situ-ations. The normative systemdesignates that in some types of socialinteraction a violent and physicallyaggressive response is either expectedor required of all members sharing inthat system of values. . . .

3. The potential resort or willingness toresort to violence in a variety of situa-tions emphasizes the penetrating anddiffusive character of this culturetheme. The number and kinds of sit-uations in which an individual usesviolence may be viewed as an index of the extent to which he has assimi-lated the values associated with violence. . . .

4. The subcultural ethos of violence may beshared by all ages in a sub-society, butthis ethos is most prominent in a limitedage group, ranging from late adolescenceto middle age. . . . [T]he known empiri-cal distribution of conduct, whichexpresses the sharing of this violencetheme, shows greatest localization,incidence, and frequency in limitedsub-groups and reflects differences inlearning about violence as a problem-solving mechanism.

5. The counter-norm is nonviolence.Violation of expected and required vio-lence is most likely to result inostracism from the group. . . .

6. The development of favorable atti-tudes toward, and the use of, violencein a subculture usually involve learnedbehavior and a process of differentiallearning, association, or identification.Not all persons exposed—even equallyexposed—to the presence of a subcul-ture of violence absorb and share inthe values in equal portions.Differential personality variables mustbe considered in an integrated social-psychological approach to an under-standing of the subcultural aspects ofviolence. . . .

7. The use of violence in a subculture is notnecessarily viewed as illicit conduct andthe users therefore do not have to dealwith feelings of guilt about their aggres-sion. Violence can become a part of thelife style, the theme of solving difficultproblems or problem situations. . . .[W]hen the attacked see their assaultersas agents of the same kind of aggressionthey themselves represent, violent retalia-tion is readily legitimized by a situation-ally specific rationale, as well as by the generally normative supports for violence. . . .

R E V I E W Q U E S T I O N S

1. What do Wolfgang and Ferracuti claim regarding two categories of homicide? What do they say regard-ing these two types of homicide (e.g., what percentage of homicides does each category make up of inthe total), and why?

2. What do the authors conclude regarding the social learning and conditioning of homicides in the sub-culture of violence?

3. What do Wolfgang and Ferracuti claim regarding the cultural context of this subculture of violence?

4. The authors provide seven conclusions in this selection. Which conclusion do you most agree with andwhy? Which conclusion do you most disagree with and why?

R E A D I N G

In this selection, Anderson presents a cultural/subcultural theory of the inner-city “ghettopoor.” He presents a model that emphasizes the interpersonal aggression of such individu-als, in which the main factor of the code is respect. An individual must maintain this respect,at all costs, even if it means resorting to lethal violence to maintain such status, especially ifthe individual has been “dissed” in public. Anderson goes further in this selection by iden-tifying both “decent” and “street” families, which provides more insight into the environ-ment and culture of inner-city youths. Anderson also provides some insightful examplesregarding the way that individuals try (or campaign) for respect in their communities.Ultimately, it is about being seen as a “man” among their peers, and acquiring such “man-hood” is seen as vitally important among this group.

The Code of the Streets

Elijah Anderson

426 CRIMINOLOGICAL THEORY: A TEXT/READER

Of all the problems besetting the poor inner-city black community, none is more pressingthan that of interpersonal violence and aggres-sion. It wreaks havoc daily with the lives ofcommunity residents and increasingly spillsover into downtown and residential middle-class areas. Muggings, burglaries, carjackings,and drug-related shootings, all of which mayleave their victims or innocent bystandersdead, are now common enough to concern allurban and many suburban residents. Theinclination to violence springs from the cir-cumstances of life among the ghetto poor—the lack of jobs that pay a living wage, thestigma of race, the fallout from rampant druguse and drug trafficking, and the resultingalienation and lack of hope for the future.

Simply living in such an environmentplaces young people at special risk of fallingvictim to aggressive behavior. Although there

are often forces in the community which cancounteract the negative influences, by far themost powerful being a strong, loving, “decent”(as inner-city residents put it) family commit-ted to middle-class values, the despair is perva-sive enough to have spawned an oppositionalculture, that of “the streets,” whose norms areoften consciously opposed to those of main-stream society. These two orientations—decent and street—socially organize thecommunity, and their coexistence has impor-tant consequences for residents, particularlychildren growing up in the inner city. Aboveall, this environment means that even young-sters whose home lives reflect mainstream val-ues—and the majority of homes in thecommunity do—must be able to handle them-selves in a street-oriented environment.

This is because the street culture hasevolved what may be called a code of the

SOURCE: Elijah Anderson, The Code of the Streets: Decency, Violence, and the Moral Life of the Inner City (New York: W. W. Norton,1999). Copyright © 1999 by Elijah Anderson. Reprinted by permission of W.W. Norton & Company.

Section VI � The Code of the Streets 427

streets, which amounts to a set of informalrules governing interpersonal public behavior,including violence. The rules prescribe both aproper comportment and a proper way torespond if challenged. They regulate the use ofviolence and so allow those who are inclined toaggression to precipitate violent encounters inan approved way. The rules have been estab-lished and are enforced mainly by the street-oriented, but on the streets the distinctionbetween street and decent is often irrelevant;everybody knows that if the rules are violated,there are penalties. Knowledge of the code isthus largely defensive; it is literally necessaryfor operating in public. Therefore, eventhough families with a decency orientation areusually opposed to the values of the code, theyoften reluctantly encourage their children’sfamiliarity with it to enable them to negotiatethe inner-city environment.

At the heart of the code is the issue ofrespect—loosely defined as being treated “right,”or granted the deference one deserves. However,in the troublesome public environment of theinner city, as people increasingly feel buffeted byforces beyond their control, what one deserves inthe way of respect becomes more and moreproblematic and uncertain. This in turn furtheropens the issue of respect to sometimes intenseinterpersonal negotiation. In the street culture,especially among young people, respect isviewed as almost an external entity that is hard-won but easily lost, and so must constantly beguarded. The rules of the code in fact provide aframework for negotiating respect. The personwhose very appearance—including his clothing,demeanor, and way of moving—deters trans-gressions feels that he possesses, and may beconsidered by others to possess, a measure ofrespect. With the right amount of respect, forinstance, he can avoid “being bothered” in pub-lic. If he is bothered, not only may he be in phys-ical danger but he has been disgraced or “dissed”(disrespected). Many of the forms that dissingcan take might seem petty to middle-class peo-ple (maintaining eye contact for too long, for

example), but to those invested in the streetcode, these actions become serious indicationsof the other person’s intentions. Consequently,such people become very sensitive to advancesand slights, which could well serve as warningsof imminent physical confrontation.

This hard reality can be traced to the pro-found sense of alienation from mainstreamsociety and its institutions felt by many poorinner-city black people, particularly the young.The code of the streets is actually a culturaladaptation to a profound lack of faith in thepolice and the judicial system. The police aremost often seen as representing the dominantwhite society and not caring to protect inner-city residents. When called, they may notrespond, which is one reason many residentsfeel they must be prepared to take extraordi-nary measures to defend themselves and theirloved ones against those who are inclined toaggression. Lack of police accountability has infact been incorporated into the status system:the person who is believed capable of “takingcare of himself” is accorded a certain defer-ence, which translates into a sense of physicaland psychological control. Thus the street codeemerges where the influence of the police endsand personal responsibility for one’s safety isfelt to begin. Exacerbated by the proliferationof drugs and easy access to guns, this volatilesituation results in the ability of the street-ori-ented minority (or those who effectively “gofor bad”) to dominate the public spaces.

yy Decent and Street FamiliesAlthough almost everyone in poor inner-cityneighborhoods is struggling financially andtherefore feels a certain distance from the restof America, the decent and the street family ina real sense represent two poles of value orien-tation, two contrasting conceptual categories.The labels “decent” and “street,” which the res-idents themselves use, amount to evaluativejudgments that confer status on local residents.

The labeling is often the result of a social con-test among individuals and families of theneighborhood. Individuals of the two orienta-tions often coexist in the same extended fam-ily. Decent residents judge themselves to be sowhile judging others to be of the street, andstreet individuals often present themselves asdecent, drawing distinctions between them-selves and other people. In addition, there isquite a bit of circumstantial behavior—that is,one person may at different times exhibit bothdecent and street orientations, depending onthe circumstances. Although these designa-tions result from so much social jockeying,there do exist concrete features that defineeach conceptual category.

Generally, so-called decent families tendto accept mainstream values more fully andattempt to instill them in their children.Whether married couples with children or sin-gle-parent (usually female) households, theyare generally “working poor” and so tend to bebetter off financially than their street-orientedneighbors. They value hard work and self-reliance and are willing to sacrifice for theirchildren. Because they have a certain amountof faith in mainstream society, they harborhopes for a better future for their children, ifnot for themselves. Many of them go to churchand take a strong interest in their children’sschooling. Rather than dwelling on the realhardships and inequities facing them, manysuch decent people, particularly the increasingnumber of grandmothers raising grandchil-dren, see their difficult situation as a test fromGod and derive great support from their faithand from the church community.

Extremely aware of the problematic andoften dangerous environment in which theyreside, decent parents tend to be strict in theirchild-rearing practices, encouraging childrento respect authority and walk a straight moralline. They have an almost obsessive concernabout trouble of any kind and remind theirchildren to be on the lookout for people andsituations that might lead to it. At the same

time, they are themselves polite and consider-ate of others, and teach their children to be thesame way. At home, at work, and in church,they strive hard to maintain a positive mentalattitude and a spirit of cooperation.

So-called street parents, in contrast, oftenshow a lack of consideration for other peopleand have a rather superficial sense of family andcommunity. Though they may love their chil-dren, many of them are unable to cope with thephysical and emotional demands of parenthood,and find it difficult to reconcile their needs withthose of their children. These families, who aremore fully invested in the code of the streetsthan the decent people are, may aggressivelysocialize their children into it in a normativeway. They believe in the code and judge them-selves and others according to its values.

In fact the overwhelming majority of fam-ilies in the inner-city community try to approx-imate the decent-family model, but there aremany others who clearly represent the worstfears of the decent family. Not only are theirfinancial resources extremely limited, but whatlittle they have may easily be misused. The livesof the street-oriented are often marked by dis-organization. In the most desperate circum-stances people frequently have a limitedunderstanding of priorities and consequences,and so frustrations mount over bills, food, and,at times, drink, cigarettes, and drugs. Sometend toward self-destructive behavior; manystreet-oriented women are crack-addicted (“onthe pipe”), alcoholic, or involved in compli-cated relationships with men who abuse them.In addition, the seeming intractability of theirsituation, caused in large part by the lack ofwell-paying jobs and the persistence of racialdiscrimination, has engendered deep-seatedbitterness and anger in many of the most des-perate and poorest blacks, especially youngpeople. The need both to exercise a measure ofcontrol and to lash out at somebody is oftenreflected in the adults’ relations with their children. At the least, the frustrations of persistent poverty shorten the fuse in such

428 CRIMINOLOGICAL THEORY: A TEXT/READER

Section VI � The Code of the Streets 429

people—contributing to a lack of patience withanyone, child or adult, who irritates them.

In these circumstances a woman—or aman, although men are less consistently pre-sent in children’s lives—can be quite aggressivewith children, yelling at and striking them forthe least little infraction of the rules she has setdown. Often little if any serious explanationfollows the verbal and physical punishment.This response teaches children a particular les-son. They learn that to solve any kind of inter-personal problem one must quickly resort tohitting or other violent behavior. Actual peaceand quiet, and also the appearance of calm,respectful children conveyed to her neighborsand friends, are often what the young mothermost desires, but at times she will be veryaggressive in trying to get them. Thus she maybe quick to beat her children, especially if theydefy her law, not because she hates them butbecause this is the way she knows to controlthem. In fact, many street-oriented womenlove their children dearly. Many mothers in thecommunity subscribe to the notion that thereis a “devil in the boy” that must be beaten outof him or that socially “fast girls need to bewhupped.” Thus, much of what borders onchild abuse in the view of social authorities isacceptable parental punishment in the view ofthese mothers.

Many street-oriented women are sporadicmothers whose children learn to fend forthemselves when necessary, foraging for foodand money any way they can get it. The chil-dren are sometimes employed by drug dealersor become addicted themselves. These childrenof the street, growing up with little supervi-sion, are said to “come up hard.” They oftenlearn to fight at an early age, sometimes usingshort-tempered adults around them as rolemodels. The street-oriented home may befraught with anger, verbal disputes, physicalaggression, and even mayhem. The childrenobserve these goings-on, learning the lessonthat might makes right. They quickly learn tohit those who cross them, and the dog-eat-dog

mentality prevails. In order to survive, to pro-tect oneself, it is necessary to marshal innerresources and be ready to deal with adversity ina hands-on way. In these circumstances physi-cal prowess takes on great significance.

In some of the most desperate cases, astreet-oriented mother may simply leave heryoung children alone and unattended whileshe goes out. The most irresponsible womencan be found at local bars and crack houses,getting high and socializing with other adults.Sometimes a troubled woman will leave veryyoung children alone for days at a time.Reports of crack addicts abandoning theirchildren have become common in drug-infested inner-city communities. Neighbors orrelatives discover the abandoned children,often hungry and distraught over the absenceof their mother. After repeated absences, afriend or relative, particularly a grandmother,will often step in to care for the young chil-dren, sometimes petitioning the authorities tosend her, as guardian of the children, themother’s welfare check, if the mother gets one.By this time, however, the children may wellhave learned the first lesson of the streets: sur-vival itself, let alone respect, cannot be takenfor granted; you have to fight for your place inthe world.

yy Campaigning for RespectThese realities of inner-city life are largelyabsorbed on the streets. At an early age, ofteneven before they start school, children fromstreet-oriented homes gravitate to the streets,where they “hang”—socialize with their peers.Children from these generally permissivehomes have a great deal of latitude and areallowed to “rip and run” up and down thestreet. They often come home from school, puttheir books down, and go right back out thedoor. On school nights eight- and nine-year-olds remain out until nine or ten o’clock (andteenagers typically come in whenever they

want to). On the streets they play in groupsthat often become the source of their primarysocial bonds. Children from decent homestend to be more carefully supervised and arethus likely to have curfews and to be taughthow to stay out of trouble.

When decent and street kids cometogether, a kind of social shuffle occurs inwhich children have a chance to go either way.Tension builds as a child comes to realize thathe must choose an orientation. The kind ofhome he comes from influences but does notdetermine the way he will ultimately turnout—although it is unlikely that a child from athoroughly street-oriented family will easilyabsorb decent values on the streets. Youthswho emerge from street-oriented families butdevelop a decency orientation almost alwayslearn those values in another setting—inschool, in a youth group, in church. Often it isthe result of their involvement with a caring“old head” (adult role model).

In the street, through their play, childrenpour their individual life experiences into acommon knowledge pool, affirming, confirm-ing, and elaborating on what they haveobserved in the home and matching their skillsagainst those of others. And they learn to fight.Even small children test one another, pushingand shoving, and are ready to hit other chil-dren over circumstances not to their liking. Inturn, they are readily hit by other children, andthe child who is toughest prevails. Thus theviolent resolution of disputes, the hitting andcursing, gains social reinforcement. The childin effect is initiated into a system that is reallya way of campaigning for respect.

In addition, younger children witness thedisputes of older children, which are oftenresolved through cursing and abusive talk, ifnot aggression or outright violence. They seethat one child succumbs to the greater physicaland mental abilities of the other. They are alsoalert and attentive witnesses to the verbal andphysical fights of adults, after which they com-pare notes and share their interpretations of

the event. In almost every case the victor is theperson who physically won the altercation, andthis person often enjoys the esteem and respectof onlookers. These experiences reinforce thelessons the children have learned at home:might makes right, and toughness is a virtue,while humility is not. In effect they learn thesocial meaning of fighting. When it is leftvirtually unchallenged, this understandingbecomes an ever more important part of thechild’s working conception of the world. Overtime the code of the streets becomes refined.

Those street-oriented adults with whomchildren come in contact—including mothers,fathers, brothers, sisters, boyfriends, cousins,neighbors, and friends—help them along informing this understanding by verbalizing themessages they are getting through experience:“Watch your back.” “Protect yourself.” “Don’tpunk out.” “If somebody messes with you, yougot to pay them back.” “If someone disses you,you got to straighten them out.” Many parentsactually impose sanctions if a child is not suffi-ciently aggressive. For example, if a child loses afight and comes home upset, the parent mightrespond, “Don’t you come in here crying thatsomebody beat you up; you better get back outthere and whup his ass. I didn’t raise no punks!Get back out there and whup his ass. If youdon’t whup his ass, I’ll whup your ass when youcome home.” Thus, the child obtains reinforce-ment for being tough and showing nerve.

While fighting, some children cry asthough they are doing something they areambivalent about. The fight may be againsttheir wishes, yet they may feel constrained tofight or face the consequences—not just frompeers but also from caretakers or parents, whomay administer another beating if they backdown. Some adults recall receiving such lessonsfrom their own parents and justify repeatingthem to their children as a way to toughenthem up. Looking capable of taking care ofoneself as a form of self defense is a dominanttheme among both street-oriented and decentadults who worry about the safety of their

430 CRIMINOLOGICAL THEORY: A TEXT/READER

Section VI � The Code of the Streets 431

children. There is thus at times a convergencein their child-rearing practices, although therationales behind them may differ.

yy Self-Image Based on “Juice”

By the time they are teenagers, most youthshave either internalized the code of the streetsor at least learned the need to comport them-selves in accordance with its rules, whichchiefly have to do with interpersonal commu-nication. The code revolves around the presen-tation of self. Its basic requirement is thedisplay of a certain predisposition to violence.Accordingly, one’s bearing must send theunmistakable, if sometimes subtle, message to“the next person” in public that one is capableof violence and mayhem when the situationrequires it, that one can take care of oneself.The nature of this communication is largelydetermined by the demands of the circum-stances but can include facial expressions, gait,and verbal expressions—all of which aregeared mainly to deterring aggression. Physicalappearance, including clothes, jewelry, andgrooming, also plays an important part in howa person is viewed; to be respected, it is impor-tant to have the right look.

Even so, there are no guarantees againstchallenges, because there are always peoplearound looking for a fight to increase their shareof respect—or “juice,” as it is sometimes called onthe street. Moreover, if a person is assaulted, it isimportant, not only in the eyes of his opponentbut also in the eyes of his “running buddies,” forhim to avenge himself. Otherwise he risks being“tried” (challenged) or “moved on” by any num-ber of others. To maintain his honor he mustshow he is not someone to be “messed with” or“dissed.” In general, the person must “keep him-self straight” by managing his position of respectamong others; this involves in part his self-image,which is shaped by what he thinks others arethinking of him in relation to his peers.

Objects play an important and compli-cated role in establishing self-image. Jackets,sneakers, gold jewelry reflect not just a person’staste, which tends to be tightly regulatedamong adolescents of all social classes, but alsoa willingness to possess things that may requiredefending. A boy wearing a fashionable,expensive jacket, for example, is vulnerable toattack by another who covets the jacket andeither cannot afford to buy one or wants theadded satisfaction of depriving someone elseof his. However, if they boy forgoes the desir-able jacket and wears one that isn’t “hip,” heruns the risk of being teased and possibly evenassaulted as an unworthy person. To beallowed to hang with certain prestigiouscrowds, a boy must wear a different set ofexpensive clothes—sneakers and athleticsuit—every day. Not to be able to do so mightmake him appear socially deficient. The youthcomes to covet such items—especially when hesees easy prey wearing them.

In acquiring valued things, therefore, aperson shores up his identity—but since it isan identity based on having things, it is highlyprecarious. This very precariousness gives aheightened sense of urgency to staying evenwith peers, with whom the person is actuallycompeting. Young men and women who areable to command respect through their pre-sentation of self—by allowing their posses-sions and their body language to speak forthem—may not have to campaign for regardbut may, rather, gain it by the force of theirmanner. Those who are unable to commandrespect in this way must actively campaign forit—and are thus particularly alive to slights.

One way of campaigning for status is bytaking the possessions of others. In this context,seemingly ordinary objects can become trophiesimbued with symbolic value that far exceedstheir monetary worth. Possession of the trophycan symbolize the ability to violate somebody—to “get in his face,” to take something of valuefrom him, to “dis” him, and thus to enhanceone’s own worth by stealing someone else’s. The

trophy does not have to be something material.It can be another person’s sense of honor,snatched away with a derogatory remark. It canbe the outcome of a fight. It can be the imposi-tion of a certain standard, such as a girl’s gettingherself recognized as the most beautiful.Material things, however, fit easily into the pat-tern. Sneakers, a pistol, even somebody else’sgirlfriend, can become a trophy. When a personcan take something from another and thenflaunt it, he gains a certain regard by being theowner, or the controller, of that thing. But thisdisplay of ownership can then provoke otherpeople to challenge him. This game of who con-trols what is thus constantly being played out oninner-city streets, and the trophy—extrinsic orintrinsic, tangible or intangible—identifies thecurrent winner.

An important aspect of this often violentgive-and-take is its zero-sum quality. That is, theextent to which one person can raise himself updepends on his ability to put another persondown. This underscores the alienation that per-meates the inner-city ghetto community. Thereis a generalized sense that very little respect is tobe had, and therefore everyone competes to getwhat affirmation he can of the little that is avail-able. The craving for respect that results givespeople thin skins. Shows of deference by otherscan be highly soothing, contributing to a senseof security, comfort, self-confidence, and self-respect. Transgressions by others which gounanswered diminish these feelings and arebelieved to encourage further transgressions.Hence one must be ever vigilant against thetransgressions of others or even appearing as iftransgressions will be tolerated. Among youngpeople, whose sense of self-esteem is particu-larly vulnerable, there is an especially height-ened concern with being disrespected. Manyinner-city young men in particular crave respectto such a degree that they will risk their lives toattain and maintain it.

The issue of respect is thus closely tied towhether a person has an inclination to be vio-lent, even as a victim. In the wider society

people may not feel required to retaliate phys-ically after an attack, even though they areaware that they have been degraded or takenadvantage of. They may feel a great need todefend themselves during an attack, or tobehave in such a way as to deter aggression(middle-class people certainly can and dobecome victims of street-oriented youths), butthey are much more likely than street-orientedpeople to feel that they can walk away from apossible altercation with their self-esteemintact. Some people may even have thestrength of character to flee, without anythought that their self-respect or esteem willbe diminished.

In impoverished inner-city black commu-nities, however, particularly among youngmales and perhaps increasingly among females,such flight would be extremely difficult. To runaway would likely leave one’s self-esteem in tat-ters. Hence people often feel constrained notonly to stand up and at least attempt to resistduring an assault but also to “pay back”—toseek revenge—after a successful assault on theirperson. This may include going to get a weaponor even getting relatives involved. Their veryidentity and self-respect, their honor, is oftenintricately tied up with the way they performon the streets during and after such encounters.This outlook reflects the circumscribed oppor-tunities of the inner-city poor. Generally peo-ple outside the ghetto have other ways ofgaining status and regard, and thus do not feelso dependent on such physical displays.

yy By Trial of ManhoodOn the street, among males these concernsabout things and identity have come to beexpressed in the concept of “manhood.”Manhood in the inner city means taking theprerogatives of men with respect to strangers,other men, and women—being distinguishedas a man. It implies physicality and a certainruthlessness. Regard and respect are associated

432 CRIMINOLOGICAL THEORY: A TEXT/READER

Section VI � The Code of the Streets 433

with this concept in large part because of itspractical application: if others have little or noregard for a person’s manhood, his very lifeand those of this loved ones could be in jeop-ardy. But there is a chicken-and-egg aspect tothis situation: one’s physical safety is morelikely to be jeopardized in public because man-hood is associated with respect. In otherwords, an existential link has been createdbetween the idea of manhood and one’s self-esteem, so that it has become hard to say whichis primary. For many inner-city youths, man-hood and respect are flip sides of the samecoin; physical and psychological well-being areinseparable, and both require a sense of con-trol, of being in charge.

The operating assumption is that a man,especially a real man, knows what other menknow—the code of the streets. And if one isnot a real man, one is somehow diminished asa person, and there are certain valued thingsone simply does not deserve. There is thusbelieved to be a certain justice to the code,since it is considered that everyone has theopportunity to know it. Implicit in this is thateverybody is held responsible for being famil-iar with the code. If the victim of a mugging,for example, does not know the code and soresponds “wrong,” the perpetrator may feeljustified even in killing him and may feel noremorse. He may think, “Too bad, but it’s hisfault. He should have known better.”

So when a person ventures outside, hemust adopt the code—a kind of shield,really—to prevent others from “messing with”him. In these circumstances it is easy for peo-ple to think they are being tried or tested byothers even when this is not the case. For it issensed that something extremely valuable is atstake in every interaction, and people areencouraged to rise to the occasion, particularlywith strangers. For people who are unfamiliarwith the code— generally people who live out-side the inner city—the concern with respectin the most ordinary interactions can befrightening and incomprehensible. But for

those who are invested in the code, the clearobject of their demeanor is to discouragestrangers from even thinking about testingtheir manhood. And the sense of power thatattends the ability to deter others can be allur-ing even to those who know the code withoutbeing heavily invested in it—the decent inner-city youths. Thus a boy who has been leading abasically decent life can, in trying circum-stances, suddenly resort to deadly force.

Central to the issue of manhood is thewidespread belief that one of the most effectiveways of gaining respect is to manifest “nerve.”Nerve is shown when one takes another per-son’s possessions (the more valuable the bet-ter), “messes with” someone’s woman, throwsthe first punch, “gets in someone’s face,” orpulls a trigger. Its proper display helps on thespot to check others who would violate one’sperson and also helps to build a reputationthat works to prevent future challenges. Butsince such a show of nerve is a forceful expres-sion of disrespect toward the person on thereceiving end, the victim may be greatlyoffended and seek to retaliate with equal orgreater force. A display of nerve, therefore, caneasily provoke a life-threatening response, andthe background knowledge of that possibilityhas often been incorporated into the conceptof nerve.

True nerve exposes a lack of fear of dying.Many feel that it is acceptable to risk dyingover the principle of respect. In fact, amongthe hard-core street-oriented, the clear risk ofviolent death may be preferable to being“dissed” by another. The youths who haveinternalized this attitude and convincingly dis-play it in their public bearing are among themost threatening people of all, for it is com-monly assumed that they fear no man. As thepeople of the community say, “They are thebaddest dudes on the street.” They often leadan existential life that may acquire meaningonly when they are faced with the possibility ofimminent death. Not to be afraid to die is byimplication to have few compunctions about

434 CRIMINOLOGICAL THEORY: A TEXT/READER

taking another’s life. Not to be afraid to die isthe quid pro quo of being able to take some-body else’s life—for the right reasons, if the sit-uation demands it. When others believe this isone’s position, it gives one a real sense ofpower on the streets. Such credibility is whatmany inner-city youths strive to achieve,whether they are decent or street-oriented,both because of its practical defensive valueand because of the positive way it makes themfeel about themselves. The difference betweenthe decent and the street-oriented youth isoften that the decent youth makes a consciousdecision to appear tough and manly; inanother setting—with teachers, say, or at hispart-time job—he can be polite and deferen-tial. The street-oriented youth, on the otherhand, has made the concept of manhood apart of his very identity; he has difficultymanipulating it—it often controls him.

yy Girls and BoysIncreasingly, teenage girls are mimicking theboys and trying to have their own version of“manhood.” Their goal is the same—to getrespect, to be recognized as capable of settingor maintaining a certain standard. They try toachieve this end in the ways that have beenestablished by the boys, including posturing,abusive language, and the use of violence toresolve disputes, but the issues for the girls aredifferent. Although conflicts over turf and sta-tus exist among the girls, the majority of dis-putes seem rooted in assessments of beauty(which girl in a group is “the cutest”), compe-tition over boyfriends, and attempts to regu-late other people’s knowledge of and opinionsabout a girl’s behavior or that of someoneclose to her, especially her mother.

A major cause of conflicts among girls is“he say, she say.” This practice begins in theearly school years and continues through highschool. It occurs when “people,” particularlygirls, talk about others, thus putting their

“business in the streets.” Usually one girl willsay something negative about another in thegroup, most often behind the person’s back.The remark will then get back to the persontalked about. She may retaliate or her friendsmay feel required to “take up for” her. Inessence this is a form of group gossiping inwhich individuals are negatively assessed andevaluated. As with much gossip, the things saidmay or may not be true, but the point is thatsuch imputations can cast aspersions on a per-son’s good name. The accused is required todefend herself against the slander, which canresult in arguments and fights, often over littleof real substance. Here again is the problem oflow self-esteem, which encourages youngstersto be highly sensitive to slights and to be vul-nerable to feeling easily “dissed.” To avenge thedissing, a fight is usually necessary.

Because boys are believed to control vio-lence, girls tend to defer to them in situations ofconflict. Often if a girl is attacked or feelsslighted, she will get a brother, uncle, or cousinto do her fighting for her. Increasingly, however,girls are doing their own fighting and are evenasking their male relatives to teach them how tofight. Some girls form groups that attack othergirls or take things from them. A hard-core seg-ment of inner-city girls inclined toward violenceseems to be developing. As one thirteen-year-oldgirl in a detention center for youths who havecommitted violent acts told me, “To get peopleto leave you alone, you gotta fight. Talking don’talways get you out of stuff.” One major differ-ence between girls and boys: girls rarely useguns. Their fights are therefore not life-or-deathstruggles. Girls are not often willing to put theirlives on the line for “manhood.” The ultimateform of respect on the male-dominated inner-city street is thus reserved for men.

yy “Going for Bad”In the most fearsome youths, such a cavalierattitude toward death grows out of a very

Section VI � The Code of the Streets 435

limited view of life. Many are uncertain abouthow long they are going to live and believethey could die violently at any time. Theyaccept this fate; they live on the edge. Theirmanner conveys the message that nothingintimidates them; whatever turn theencounter takes, they maintain their attack—rather like a pit bull, whose spirit many suchboys admire. The demonstration of suchtenacity “shows heart” and earns their respect.

This fearlessness has implications for lawenforcement. Many street-oriented boys aremuch more concerned about the threat of“justice” at the hands of a peer than at thehands of the police. Moreover, many feel notonly that they have little to lose by going toprison but that they have something to gain.The toughening-up one experiences inprison can actually enhance one’s reputationon the streets. Hence the system loses influ-ence over the hard core who are without jobs,with little perceptible stake in the system. Ifmainstream society has done nothing forthem, they counter by making sure it can donothing to them.

At the same time, however, a competingview maintains that true nerve consists inbacking down, walking away from a fight, andgoing on with one’s business. One fights onlyin self-defense. This view emerges from thedecent philosophy that life is precious, and it isan important part of the socialization processcommon in decent homes. It discourages vio-lence as the primary means of resolving dis-putes and encourages youngsters to acceptnonviolence and talk as confrontational strate-gies. But “if the deal goes down,” self-defense isgreatly encouraged. When there is enough pos-itive support for this orientation, either in thehome or among one’s peers, then nonviolencehas a chance to prevail. But it prevails at thecost of relinquishing a claim to being bad andtough, and therefore sets a young person up as,at the very least, alienated from street-orientedpeers and quite possibly a target of derision oreven violence.

Although the nonviolent orientationrarely overcomes the impulse to strike back inan encounter, it does introduce a certain con-fusion, and so can prompt a measure of soul-searching, or even profound ambivalence. Didthe person back down with his respect intactor did he back down only to be judged a“punk”—a person lacking manhood? Shouldhe or she have acted? Should he or she have hitthe other person in the mouth? These ques-tions beset many young men and women dur-ing public confrontations. What is the “right”thing to do? In the quest for honor, respect,and local status—which few young people areuninterested in—common sense most oftenprevails, which leads many to opt for the toughapproach, enacting their own particular ver-sions of the display of nerve. The presentationof oneself as rough and tough is very oftenquite acceptable until one is tested. And thenthat presentation may help the person pass thetest, because it will cause fewer questions to beasked about what he did and why. It is hard fora person to explain why he lost the fight or whyhe backed down. Hence many will strive toappear to “go for bad,” while hoping they willnever be tested. But when they are tested, theoutcome of the situation may quickly be out oftheir hands, as they become wrapped up in thecircumstances of the moment.

yy An Oppositional CultureThe attitudes of the wider society are deeplyimplicated in the code of the streets. Most peo-ple in inner-city communities are not totallyinvested in the code, but the significant minor-ity of hard-core street youths who are have tomaintain the code in order to establish reputa-tions, because they have—or feel they have—few other ways to assert themselves. For theseyoung people the standards of the street codeare the only game in town. The extent to whichsome children—particularly those who throughupbringing have become most alienated and

those lacking in strong and conventional socialsupport—experience, feel, and internalizeracist rejection and contempt from main-stream society may strongly encourage themto express contempt for the more conventionalsociety in turn. In dealing with this contemptand rejection, some youngsters will con-sciously invest themselves and their consider-able mental resources in what amounts to anoppositional culture to preserve themselvesand their self-respect. Once they do, anyrespect they might be able to garner in thewider system pales in comparison with therespect available in the local system; thus theyoften lose interest in even attempting to nego-tiate the mainstream system.

At the same time, many less alienatedyoung blacks have assumed a street-orienteddemeanor as a way of expressing their black-ness while really embracing a much moremoderate way of life; they, too, want a nonvi-olent setting in which to live and raise a fam-ily. These decent people are trying hard to be

part of the mainstream culture, but theracism, real and perceived, that they encounterhelps to legitimate the oppositional culture.And so on occasion they adopt street behav-ior. In fact, depending on the demands of thesituation, many people in the community slipback and forth between decent and streetbehavior.

A vicious cycle has thus been formed. Thehopelessness and alienation many younginner-city black men and women feel, largelyas a result of endemic joblessness and persis-tent racism, fuels the violence they engage in.This violence serves to confirm the negativefeelings many whites and some middle-classblacks harbor toward the ghetto poor, furtherlegitimating the oppositional culture and thecode of the streets in the eyes of many pooryoung blacks. Unless this cycle is broken, atti-tudes on both sides will become increasinglyentrenched, and the violence, which claimsvictims, black and white, poor and affluent,will only escalate.

436 CRIMINOLOGICAL THEORY: A TEXT/READER

R E V I E W Q U E S T I O N S

1. What type of individuals is Anderson discussing? Do you personally know any individuals who fit thisdescription?

2. Which of Anderson’s explanations do you find most credible? Which portions do you find least credible?

3. Given that Anderson’s description is valid for inner-city, urban youth, to what extent do you feel thatthis explains crime rates in the United States? Which significant portion do you feel that this model doesnot cover?